{ "cells": [ { "cell_type": "markdown", "metadata": {}, "source": [ "# Past Earth Network Emulator Workshop: Lab 1" ] }, { "cell_type": "markdown", "metadata": {}, "source": [ "This tutorial is based on ones written by [Alan Saul](http://www.alansaul.com/), James Hensman, Neil Lawrence, and Nicolas Durrande, with additions by Richard Wilkinson. In it, we will run through some of the basic features of GPy. We will focus on three aspects of GPs: the kernel/covariance function, how to generate random sample paths, and how to do GP regression.\n" ] }, { "cell_type": "markdown", "metadata": {}, "source": [ "We assume that GPy is already installed on your machine. You can get instructions on how to install GPy from the [SheffieldML github page](https://github.com/SheffieldML/GPy). The online documentation for GPy is available from [this page](http://gpy.readthedocs.org/en/latest/).\n", "\n", "To start off, we must first tell the Jupyer notebook that we want the plots to appear inline and import the libraries we will need:" ] }, { "cell_type": "code", "execution_count": 1, "metadata": { "collapsed": false }, "outputs": [], "source": [ "%matplotlib inline\n", "import numpy as np\n", "from matplotlib import pyplot as plt\n", "import GPy" ] }, { "cell_type": "markdown", "metadata": {}, "source": [ "## 1. The Covariance Function" ] }, { "cell_type": "markdown", "metadata": {}, "source": [ "The covariance function is the single most important aspect of a GP. We can use any function we like as long as it is positive semi-definite. We usually build a suitable covariance function by combining a small number of well-known covariance functions. However, it is possible to add your own kernels to GPy without too much trouble. A good source of information on covariance functions is available in the [kernel cookbook](http://www.cs.toronto.edu/~duvenaud/cookbook/index.html)\n", "\n", "Let's start by defining an exponentiated quadratic covariance function (also known as squared exponential or rbf - radial basis function - or Gaussian) in one dimension:" ] }, { "cell_type": "code", "execution_count": 2, "metadata": { "collapsed": false }, "outputs": [], "source": [ "d = 1 # input dimension\n", "var = 1. # variance\n", "theta = 0.2 # lengthscale\n", "k = GPy.kern.RBF(d, variance=var, lengthscale=theta)" ] }, { "cell_type": "markdown", "metadata": {}, "source": [ "A summary of the kernel can be obtained using the command `print k`. " ] }, { "cell_type": "code", "execution_count": 3, "metadata": { "collapsed": false }, "outputs": [ { "name": "stdout", "output_type": "stream", "text": [ " \u001b[1mrbf. \u001b[0;0m | value | constraints | priors\n", " \u001b[1mvariance \u001b[0;0m | 1.0 | +ve | \n", " \u001b[1mlengthscale\u001b[0;0m | 0.2 | +ve | \n" ] } ], "source": [ "print(k)" ] }, { "cell_type": "markdown", "metadata": {}, "source": [ "It is also possible to plot the kernel as a function of one of its inputs (whilst fixing the other) with `k.plot()`. \n", "\n", "*Note*: if you need help with a command in ipython notebook, then you can get it at any time by typing a question mark after the command, e.g. `k.plot?`" ] }, { "cell_type": "code", "execution_count": 4, "metadata": { "collapsed": false }, "outputs": [ { "data": { "text/plain": [ "" ] }, "execution_count": 4, "metadata": {}, "output_type": "execute_result" }, { "name": "stderr", "output_type": "stream", "text": [ " /Users/pmzrdw/anaconda/lib/python3.5/site-packages/matplotlib/figure.py:1742: UserWarning:This figure includes Axes that are not compatible with tight_layout, so its results might be incorrect.\n" ] }, { "data": { "image/png": "iVBORw0KGgoAAAANSUhEUgAAAk4AAAGGCAYAAACNCg6xAAAABHNCSVQICAgIfAhkiAAAAAlwSFlz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\nTgYnSZKknAxOkiRJOf1/58ZqXH1sEacAAAAASUVORK5CYII=\n", "text/plain": [ "" ] }, "metadata": {}, "output_type": "display_data" } ], "source": [ "k.plot()" ] }, { "cell_type": "markdown", "metadata": {}, "source": [ "## Setting Covariance Function Parameters" ] }, { "cell_type": "markdown", "metadata": {}, "source": [ "The value of the covariance function parameters can be accessed and modified using `k['.*var']` where the string in brackets is a regular expression matching the parameter name as it appears in `print k`. You can also access each parameter directly:" ] }, { "cell_type": "code", "execution_count": 5, "metadata": { "collapsed": false }, "outputs": [ { "name": "stdout", "output_type": "stream", "text": [ " \u001b[1mindex\u001b[0;0m | rbf.lengthscale | constraints | priors\n", " \u001b[1m[0] \u001b[0;0m | 0.20000000 | +ve | \n" ] }, { "data": { "text/html": [ "\n", "\n", "\n", "\n", " \n", " \n", " \n", "\n", "" ], "text/plain": [ "\u001b[1mrbf.lengthscale\u001b[0;0m:\n", "Param([ 0.2])" ] }, "execution_count": 5, "metadata": {}, "output_type": "execute_result" } ], "source": [ "print(k.lengthscale)\n", "k['.*lengthscale']" ] }, { "cell_type": "markdown", "metadata": {}, "source": [ "Let's use this to get an insight into the effect of the parameters on the shape of the covariance function, by setting the lengthscale of the covariance function to different values, and plotting the resulting covariance using the `k.plot()` method." ] }, { "cell_type": "code", "execution_count": 6, "metadata": { "collapsed": false }, "outputs": [ { "data": { "text/plain": [ "" ] }, "execution_count": 6, "metadata": {}, "output_type": "execute_result" }, { "name": "stderr", "output_type": "stream", "text": [ " /Users/pmzrdw/anaconda/lib/python3.5/site-packages/matplotlib/figure.py:1742: UserWarning:This figure includes Axes that are not compatible with tight_layout, so its results might be incorrect.\n" ] }, { "data": { "image/png": "iVBORw0KGgoAAAANSUhEUgAAAk4AAAGGCAYAAACNCg6xAAAABHNCSVQICAgIfAhkiAAAAAlwSFlz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\nTgYnSZKknAxOkiRJOf1/58ZqXH1sEacAAAAASUVORK5CYII=\n", "text/plain": [ "" ] }, "metadata": {}, "output_type": "display_data" }, { "data": { "image/png": "iVBORw0KGgoAAAANSUhEUgAAAk4AAAGGCAYAAACNCg6xAAAABHNCSVQICAgIfAhkiAAAAAlwSFlz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\n4CRJklQmg5MkSVKZ/h+L0qN+J6a1TgAAAABJRU5ErkJggg==\n", "text/plain": [ "" ] }, "metadata": {}, "output_type": "display_data" }, { "data": { "image/png": "iVBORw0KGgoAAAANSUhEUgAAAk4AAAGGCAYAAACNCg6xAAAABHNCSVQICAgIfAhkiAAAAAlwSFlz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\nAElFTkSuQmCC\n", "text/plain": [ "" ] }, "metadata": {}, "output_type": "display_data" }, { "data": { "image/png": "iVBORw0KGgoAAAANSUhEUgAAAk4AAAGGCAYAAACNCg6xAAAABHNCSVQICAgIfAhkiAAAAAlwSFlz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\nQmCC\n", "text/plain": [ "" ] }, "metadata": {}, "output_type": "display_data" }, { "data": { "image/png": "iVBORw0KGgoAAAANSUhEUgAAAk4AAAGGCAYAAACNCg6xAAAABHNCSVQICAgIfAhkiAAAAAlwSFlz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\nTpIkSSX9HyDAHB2Dau8eAAAAAElFTkSuQmCC\n", "text/plain": [ "" ] }, "metadata": {}, "output_type": "display_data" } ], "source": [ "k = GPy.kern.RBF(d) # By default, the parameters are set to 1.\n", "theta = np.asarray([0.2,0.5,1.,2.,4.])\n", "for t in theta:\n", " k.lengthscale=t\n", " k.plot()\n", "plt.legend(theta)" ] }, { "cell_type": "markdown", "metadata": {}, "source": [ "### Exercise 1" ] }, { "cell_type": "markdown", "metadata": {}, "source": [ "a) What is the effect of the lengthscale parameter on the covariance function?" ] }, { "cell_type": "raw", "metadata": {}, "source": [ "# Exercise 1 a) answer" ] }, { "cell_type": "markdown", "metadata": {}, "source": [ "b) Now change the code above to investigate the effect of changing the variance parameter." ] }, { "cell_type": "code", "execution_count": 7, "metadata": { "collapsed": false }, "outputs": [], "source": [ "# Exercise 1 b) answer\n" ] }, { "cell_type": "markdown", "metadata": {}, "source": [ "## Covariance Functions in GPy" ] }, { "cell_type": "markdown", "metadata": {}, "source": [ "Many different covariance functions are already implemented in GPy. Instead of rbf, try constructing and plotting the following covariance functions: `exponential`, `Matern32`, `Matern52`, `Brownian`, `linear`, `bias`,\n", "`rbfcos`, `periodic_Matern32`, etc. Some of these covariance functions, such as `rbfcos`, are not\n", "parametrized by a variance and a lengthscale. Furthermore, not all kernels are stationary (i.e., they can’t all be written as $k ( x, y) = k ( x − y)$, see for example the Brownian\n", "covariance function). For plotting so it may be interesting to change the value of the fixed input:" ] }, { "cell_type": "code", "execution_count": 8, "metadata": { "collapsed": false }, "outputs": [ { "data": { "text/plain": [ "(-0.1, 5.1)" ] }, "execution_count": 8, "metadata": {}, "output_type": "execute_result" }, { "name": "stderr", "output_type": "stream", "text": [ " /Users/pmzrdw/anaconda/lib/python3.5/site-packages/matplotlib/figure.py:1742: UserWarning:This figure includes Axes that are not compatible with tight_layout, so its results might be incorrect.\n" ] }, { "data": { "image/png": "iVBORw0KGgoAAAANSUhEUgAAAk4AAAGGCAYAAACNCg6xAAAABHNCSVQICAgIfAhkiAAAAAlwSFlz\nAAAPYQAAD2EBqD+naQAAIABJREFUeJzt3XecXWW97/HPb2bSO4QklEDoEAjpmSjSBERAivQQEo6o\nR5FzULAhcO/xnksTRBQV9eqxhJAgglRFehEhCSkECKETSEglgfQ2M8/9YyZjiCl7JjOzdvm8X695\n6bP2Wosv7GTmO2ut/TyRUkKSJEnbVpZ1AEmSpEJhcZIkScqRxUmSJClHFidJkqQcWZwkSZJyZHGS\nJEnKkcVJkiQpRxYnSZKkHFmcJEmScmRxkiRJylHmxSkivhcRkyJiWUQsiIi7I2K/HI47MiKmRMSa\niHg9Is5vibySJKl0ZV6cgMOAnwKVwDFAK+DhiGi3pQMiog/wAPAY0B/4CfCbiDi2ucNKkqTSFfm2\nyG9EdAcWAoenlJ7Zwj4/AI5PKR2y0bbxQJeU0gktk1SSJJWafLjitKmuQAKWbGWf4cCjm2x7CPhE\nc4WSJEnKq+IUEQH8GHgmpfTKVnbtBSzYZNsCoHNEtGmufJIkqbRVZB1gE7cAfYFDm/rEEbEjcBww\nC1jT1OeXJEkFoy3QB3gopbS4IQfmTXGKiJ8BJwCHpZTmbWP3+UDPTbb1BJallNZu4ZjjgNu2L6Uk\nSSoiI4FxDTkgL4pTXWk6BTgipfReDoc8Bxy/ybbP1G3fklkAY8eO5cADD2xMTDWjSy65hJtuuinr\nGKrz9tyPuObWSaxZX8Xs58Zy+GkX8emBu2cdS5vx259dzQX/cUXWMbQZvjf5a867b/Hjq74Jdd2g\nITIvThFxCzACOBlYGREbriQtTSmtqdvnGmDXlNKGuZp+CVxU9+m63wJHA2dQe8VqS9YAHHjggQwa\nNKjp/0W0Xbp06eL7kifWra/munv/SnmX3nQAdtihG3/84Vfp2K5V1tG0GU/d9z9880unZB1Dm+F7\nk7+mTp3Kj68CGvHoTj48HP5VoDPwJDB3o6+zNtpnZ6D3hkFKaRZwIrXzPr0AXAJ8MaW06SftJDXQ\nH/42g3fmLQPgwD12oN+e3S1NklQn8ytOKaVtlreU0hc2s+1pYHCzhJJK1Ntzl/Lbv84AoLwsuHL0\ncL418RcZp5Kk/JEPV5wk5YHqmhquGjOBquoaAEYd15f9enfLOJUk5ReLk/LCiBEjso5Q8u568g1e\nevsDAHbv0Ykvnngw4HuT73x/8pfvTXHKuyVXmktEDAKmTJkyxYeQpU3MX7ySs7//AKvWVgHwy28e\nw+D9N53xQ5KKw9SpUxk8eDDA4JTS1IYc6xUnqcSllLjutkn1penUw/axNEnSFlicpBL3yPPv8o+X\n5wLQvUs7Lj59YMaJJCl/WZykEvbRirX88I+T68ffOXcondq3zjCRJOU3i5NUwn78p6l8uLx2laKj\nBvbmqIG9t3GEJJU2i5NUoibMmMdfnnsbgI7tWvHtEUMyTiRJ+c/iJJWg1WuruHbsxPrxxWcMYqeu\n7TNMJEmFweIklaBf3jeduYtXAjBovx6ccujeGSeSpMJgcZJKzIxZi7n90dcAaF1RxuWjKikri4xT\nSVJhsDhJJaSqqoarx0ygpm7i2y+d1I89enbOOJUkFQ6Lk1RCbn3kFd6Y8xEA++7WjVHH9s04kSQV\nFouTVCLeXbCM39z/EgBlEVw5upKKCr8FSFJD+F1TKgE1NYmrx0xkXVUNACOOOYC+fXbMOJUkFR6L\nk1QC7nnmTaa9sRCAXbp35CsnH5JxIkkqTBYnqcgt+mgVN985rX58+XnDaNemIsNEklS4LE5Skbt+\n/GRWrlkPwImf2IvKvjtnnEiSCpfFSSpij099jyenzQagW6c2fOPMQRknkqTCZnGSitTyVeu4ftzz\n9eNvnTOErh3bZJhIkgqfxUkqUjffNY3Fy9YA8KlDduXYIXtknEiSCp/FSSpCU15bwD1/fxOA9m0q\n+O65Q4lwWRVJ2l4WJ6nIrFlXxdW3TqwfX3TaAHrt0CHDRJJUPCxOUpH5zQMvMXvhcgD67dWdM47Y\nL+NEklQ8LE5SEXlt9hLGPjwTgIryMq4cPZyyMm/RSVJTsThJRaKquoarx0ykuiYBcMEJB7HXLl0y\nTiVJxcXiJBWJ2x97jZnvLgFgr1268G/HH5RxIkkqPhYnqQjMWbScX947HYAIuGJUJa0qyjNOJUnF\nx+IkFbiUEteOncTa9dUAnHXU/hyy904Zp5Kk4mRxkgrcA8+9zaSZ8wHo2a09F57aP+NEklS8LE5S\nAVu8bDU/vmNq/fiykcPo0LZVhokkqbhZnKQCduPtU1i2ah0Axw3bg08dsmvGiSSpuFmcpAL19PQ5\nPDL5XQC6dGjNpWcNyTiRJBU/i5NUgFasXs8Pxj1fP770rMHs0LlthokkqTRYnKQCdMvdL7Dww1UA\nDO+7M8cP3zPjRJJUGixOUoGZ/uZC7nzqdQDati7nsvOGEeGyKpLUEixOUgFZt76aq2+dSKpdVYUL\nT+nPrt07ZhtKkkqIxUkqIL97cAbvzFsGQN8+O3L20ftnnEiSSovFSSoQb77/Eb9/cAYA5WXBFaMq\nKS/zr7AktSS/60oFoLqmhqvHTKCqugaA0cf1Zb/e3TJOJUmlx+IkFYA7n3yDl99ZDMDuPTvxxc/1\nyziRJJUmi5OU5+YvXsnP736hfnzFqEratCrPMJEklS6Lk5THUkpcd9skVq+tAuC0w/dh0H49M04l\nSaXL4iTlsYeff5d/vDwXgO5d2vGfpw3MOJEklTaLk5SnPlq+hh/ePrl+/N1zh9KxfesME0mSLE5S\nnrrpT1P5aMVaAD49qDdHDuydcSJJksVJykPPzZjLXye8A0Cn9q359jlDM04kSQKLk5R3Vq1Zz7Vj\nJ9WPv37GQLp3bZdhIknSBhYnKc/86r4Xmbd4JQBD9u/JyYfunXEiSdIGFicpj8x45wNuf+w1ANq0\nKud75w0jIjJOJUnawOIk5YmqqhquGjORmpQA+PJJ/di9Z+eMU0mSNmZxkvLEmIde4c33PwJgv97d\nGHnMgRknkiRtyuIk5YFZ85bym7+8BEBZBFeOrqSiwr+ekpRv/M4sZaymJnHN2Imsr6oB4NxjD+DA\nPXbMOJUkaXMsTlLG7vn7m0x7YxEAu3bvyFdOOiTjRJKkLbE4SRla+OEqbr5rWv348lGVtG1TkWEi\nSdLWWJykjKSUuH7c86xcsx6Akz65F8MO7JVxKknS1licpIw8MXU2T02fA8AOndry9TMHZZxIkrQt\nFicpA8tWruX68c/Xj781YghdOrTJMJEkKRcWJykDN981jcXL1gBw2CG7cszg3TNOJEnKhcVJamHP\nvzqfe595C4AObSv47kiXVZGkQmFxklrQmnVVXHPrxPrxRacNpGe39hkmkiQ1hMVJakG/fuAl5ixa\nAUD/vXfi9MP3zTiRJKkhLE5SC3ntvSXc9vBMAFpVlHHF6ErKyrxFJ0mFxOIktYCq6hquGjOR6poE\nwAUnHMyeO3fJOJUkqaEsTlILGP/Yq7z63hIA9tqlC+d/tm/GiSRJjWFxkprZnIXL+dW9LwIQAVeO\nHk6rivKMU0mSGsPiJDWjlBJXj53I2vXVAJx11P7026t7xqkkSY1lcZKa0f3Pvs3kVxcA0GuH9nzt\n1P4ZJ5IkbQ+Lk9RMPli6mp/8aWr9+LKRw2jftlWGiSRJ2ysvilNEHBYR90XE+xFRExEnb2P/I+r2\n2/irOiJ6tFRmaVtu/ONklq1aB8BnK/twaL9dM04kSdpeeVGcgA7AC8DXgJTjMQnYF+hV97VzSmlh\n88STGuapF+bw6OT3AOjSoQ2XnjU440SSpKZQkXUAgJTS34C/AUTDFu1alFJa1jyppMZZsXo914+b\nVD++9OzBdOvUNsNEkqSmki9XnBojgBciYm5EPBwRn8w6kATwsz9PY+FHqwEYftDOHF/ZJ9tAkqQm\nU6jFaR7wFeB04DRgNvBkRAzINJVK3gtvLOSup94AoG3rcr43chgNu4gqScpneXGrrqFSSq8Dr2+0\naUJE7A1cApyfTSqVurXrq7n61on14wtPHcAu3TtmmEiS1NQKsjhtwSTg0G3tdMkll9Cly8fXCBsx\nYgQjRoxorlwqEb/768vMml/7yN1BfXbk7E/vl3EiSdL48eMZP378x7YtXbq00eeLlHL9EFvLiIga\n4NSU0n0NPO5hYFlK6YwtvD4ImDJlyhQGDRrUBEmlf3pzzoecd9WDVNckysuCsVcezz67dcs6liRp\nM6ZOncrgwYMBBqeUpm5r/43lxRWniOgA7EPtA98Ae0VEf2BJSml2RFwL7JJSOr9u/68D7wAzgLbA\nl4GjgGNbPLxKXnVNDVffOpHqmtpfQs4//iBLkyQVqbwoTsAQ4Alq52ZKwI112/8AXEDtPE29N9q/\ndd0+uwCrgBeBo1NKT7dUYGmDPz3xOi+/sxiAPXp25oITDs44kSSpueRFcUopPcVWPuGXUvrCJuMb\ngBuaO5e0LfMWr+CWe6bXj68YXUmbVuUZJpIkNadCnY5AylxKiWvHTmL12ioATj9iXwbu66o/klTM\nLE5SI/1t0iyemzEPgJ26tuM/Pu80YpJU7CxOUiN8uHwNN94+pX783XOH0bF96wwTSZJagsVJaoSb\n7pjC0pVrAThm8O4cMWC3jBNJklqCxUlqoGdfnsuDE2cB0Kl9a755zpBsA0mSWozFSWqAVWvWc91t\nk+rH3zhjEN27tMswkSSpJVmcpAb45b0vMm/xSgCGHNCTkw7dK+NEkqSWZHGScvTy2x9w++OvAtCm\nVTmXn1dJRGzjKElSMbE4STlYX1XNVWMmsGFpx38/qR+9e3TKNpQkqcVZnKQcjHnoFd6aW7ua9v69\nu3HusQdmnEiSlAWLk7QNs+Yt5X/+8jIA5WXBlaOHU1HuXx1JKkV+95e2oqYmcfWtE1lfVQPAyGMP\n5IA9dsg4lSQpKxYnaSvu/vsbvPDmIgB226kjX/5cv4wTSZKyZHGStmDhh6v46V3T6seXj6qkbZuK\nDBNJkrJmcZI2I6XED8Y9z8o1VQCcfOjeDD2gV8apJElZszhJm/HYlPd4evocAHbo3JavnzEw40SS\npHxgcZI2sXTlWm4YP7l+/J0RQ+ncoU2GiSRJ+cLiJG3i5junsWT5GgCO6L8bnx7UO+NEkqR8YXGS\nNjJp5nzu+8dbAHRo24rvnDvUZVUkSfUsTlKdNWuruHbsxPrxf54+gB7d2meYSJKUbyxOUp1fP/AS\ncxatAGDAPjvx+cP2zTiRJCnfWJwk4NV3l3DbIzMBaFVRxhWjKikr8xadJOnjLE4qeVXVNVw1ZgLV\nNQmAL554MH127pJxKklSPrI4qeSNe2Qmr83+EIB9du3K6OP6ZpxIkpSvLE4qabMXLuf/3f8SABFw\n5ehKWlWUZ5xKkpSvLE4qWSklrrl1ImvXVwNwzqcP4KA9u2ecSpKUzyxOKln3/+NtJr+2AICdd+zA\nV085JONEkqR8Z3FSSfpg6Wp+fOfU+vFlI4fRvm2rDBNJkgqBxUkl6cbbJ7N81ToAjq/swycP3iXj\nRJKkQmBxUsl56oXZPDrlPQC6dmzDpWcNzjiRJKlQWJxUUlasWscPxj1fP7707MF07dQ2w0SSpEJi\ncVJJ+dndL7Doo9UAfPLgXfjssD7ZBpIkFRSLk0rGtDcWctdTbwDQrk0Fl40cSoTLqkiScmdxUklY\nu76aq8dMrB9/7dT+7LxjxwwTSZIKkcVJJeG3f32ZdxcsA+DgPXfkzKP2yziRJKkQWZxU9N6c8yF/\neHAGAOVlwRWjh1Ne5h99SVLDVTTmoIhoBfQC2gOLUkpLmjSV1ESqa2q4asxEqmsSAP92/EHss2vX\njFNJkgpVzr92R0SniLgwIp4ClgGzgJnAooh4NyJ+HRFDmymn1Ch/fPx1ZsxaDECfXp254ISDM04k\nSSpkORWniLiU2qL0BeBR4FRgALAf8Ang/1B79erhiPhbROzbLGmlBpj7wQp+cc8L9eMrR1fSulV5\nhokkSYUu11t1Q4HDU0oztvD6JOC3EfFVasvVYcAbTZBPapSUEtfeNok166oBOOPIfem/T4+MU0mS\nCl1OxSmlNCLH/dYCv9yuRFITeHDiLCbMmAdAj67tuOjzAzNOJEkqBn60SEXnw+Vr+NEfp9SPv3Pu\nMDq2a5VhIklSscjpilNETG3geRNwckrp/YZHkrbPj+6YwtKVawE4ZsjuHDFgt4wTSZKKRa7POA0A\nbgRW5LBvAJcBbRobSmqsf7z0Pn+bOAuAzu1b861zhmQbSJJUVBoyj9MNKaWFuewYEd9sZB6p0Vau\nWc91t02qH3/9zEHs2LldhokkScUm1+K0J7CoAeftC8xteByp8X5xz3TmL1kFwNADenHSJ/fKOJEk\nqdjk+qm6dxty0pTS7MbFkRrnpbc/4I4nXgOgTatyLj9vGBGRcSpJUrHJ9eHwkxtx7kdSSqsbcZzU\nIOurqrlqzARS7aoqfOWUQ9itR6dsQ0mSilKut+ruaeB5E7Av8HYDj5Ma7A9/e4W35y4F4IDdd2DE\n0QdknEiSVKwa8nB4rwY8HL68kXmkBnln3lJ++9eXASgvC64cXUlFudOTSZKaR64/Yf4ANOS221hq\nFwKWmk1NTeLqMRNZX1UDwMjPHMj+u++QcSpJUjHL9eHwLzTkpCmlCxsXR8rdXU+/wfS3aj/s2btH\nJ778uX4ZJ5IkFTvvaaggzV+ykp//eVr9+PLzhtG2dUPuPEuS1HANKk4R0TcibomIaRExr+5rWt22\nvs0VUtpYSokfjHuelWuqADjlU3sz5IBeGaeSJJWCnH9Fj4jjqf103VTgXmBB3Us9gWOBqRFxSkrp\noSZPKW3k0Snv8cyLtcsg7ti5LRefPjDjRJKkUtGQexvXAT9IKf3vzbz2/Yj4PnADYHFSs1m6ci0/\nHD+5fvydEUPp3MFlESVJLaMht+r2A27byuvjqZ27SWo2P/nTVJYsXwPAkQN246hBvTNOJEkqJQ0p\nTrOAE7fy+olAg5ZmkRpi4ivzuP/Z2jlVO7RtxbdHDHVZFUlSi2rIrbr/DYyLiCOBR/n4M05HA58F\nzm3SdFKdNWuruHbspPrxxacPpEe39hkmkiSVopyLU0rpTxHxPnAx8E1gw8eY5gPPAUemlJ5r+ogS\n/Or+F3n/gxUADNy3B6cetk/GiSRJpahBE9+klJ4Fnm2mLNJmzXx3MeMeeRWA1hVlXD6qkrIyb9FJ\nklqeE2Aqr1VV1XDVmInUpATAlz7Xjz69OmecSpJUqpqsOEXENRHx26Y6nwRw26MzeX32hwDss2tX\nRn3GeVYlSdlpyjUqdgX8bLiazHsLlvHr+18CoCyCK0dXUlHhRVJJUnaarDillM5vqnNJKSWuuXUS\na9dXA3DO0ftz0J7dM04lSSp1/vquvHTfP95iyuu1M17ssmMHvnpK/4wTSZLUiOIUEbtFRMfNbG8V\nEYc3TSyVsg8+Ws2P/zS1fvy98ypp16Yp7ypLktQ4OReniNg5IiZROzv4RxExZpMCtQPwRFMHVOm5\n4fbnWbF6PQAnfmJPhh+0c8aJJEmq1ZArTtcBNUAltbOE9wWeiIhuG+3j5DraLk9Mm83jU2cD0K1T\nG75x5uCME0mS9E8NKU7HABenlCanlB4FDgXmAY9HxA51+6TGhIiIwyLivoh4PyJqIuLkHI45MiKm\nRMSaiHg9Inw4vcAtX7WO68c9Xz/+5tlD6NqxTYaJJEn6uIYUpy7AhxsGKaW1wGnULv77BNBjO3J0\nAF4AvkYO5Ssi+gAPAI8B/YGfAL+JiGO3I4My9rM/T+ODpasBOPTgXfjM0D0yTiRJ0sc15Inbt4FD\ngDc2bEgpVUXEmcCfqC0yjZJS+hvwN4DIbbn7C4G3U0rfqRu/FhGfAi4BHmlsDmVn6usL+PPTbwLQ\nrk0Fl40cRm5/FCRJajkNueL0IPDvm25MKVUBZ1J7xailDAce3WTbQ8AnWjCDmsja9dVcfevE+vFF\nnx9Arx07ZJhIkqTNa8gVpyuA9pt7oe7K0+nUzh7eEnoBCzbZtgDoHBFt6m4jqkD8zwMv8d6C5QD0\n26s7Zxy5b8aJJEnavJyLU92VpWXbeP3dpgil0vH67A8Z89ArAFSUl3HF6ErKy5yXVZKUn3IqThHx\nI+B/pZRW5rj/tcANKaUl2xNuK+YDPTfZ1hNYtq2rTZdccgldunT52LYRI0YwYsSIpk2obaquqeGq\nMROorqn9PMAXjj+IvXfpmnEqSVIxGT9+POPHj//YtqVLlzb6fJHStmcQiIhqoFdKaVFOJ41YBgxI\nKb3d4EARNcCpKaX7trLPdcDxKaX+G20bB3RNKZ2whWMGAVOmTJnCoEGDGhpLzWDcIzO5qW6G8D13\n7szYK0+gdavyjFNJkord1KlTGTx4MMDglNLUbe2/sVxv1QXwekTkOk9Tg57sjYgOwD78cwLNvSKi\nP7AkpTS77grWLhstJPxL4KKI+AHwW+Bo4Axgs6VJ+ef9D1bwi3unAxABV4webmmSJOW9XIvTFxpx\n7k0f3t6aIdTOBZXqvm6s2/4H4AJqHwbvvWHnlNKsiDgRuAm4GJgDfLFuYk7luZQS142dxJp11QCc\nccR+9N97p4xTSZK0bTkVp5TSH5ozRErpKbYyNUJK6V+KW0rpacD1OArQgxPeYcIr8wDo0a09X/v8\ngIwTSZKUm5w+vhQRnRty0ojo1Lg4KnZLlq3hR3dMqR9fNnIoHdu1yjCRJEm5y/Vz3x9GREOWVHk/\nIvZqTCAVtx/dMZmlK9cB8Jmhe3DYIbtlnEiSpNw15OHwL0XEihz39xKC/sUzL77PQ5Nqp/rq0qE1\n3zx7SMaJJElqmFyL03vAlxtw3vnA+obHUbFauWY91902qX78jTMHs0PnthkmkiSp4XJ9OLxPM+dQ\nkfvFPdNZ8OEqAIYd2IsTP7FnxokkSWo417ZQs3vxrUXc8cRrALRpVc73zhtGRGzjKEmS8o/FSc1q\nfVU1V42ZyIYJ6r96Sn9228kPXUqSCpPFSc3q9397hXfm1a4JdOAeO3DO0ftnnEiSpMazOKnZvD13\nKb/9y8sAlJcFV44eTkW5f+QkSYXLn2JqFjU1iavGTKCqugaAUcf1Zb/e3TJOJUnS9mnS4hQRu0eE\nK7WKO596nZfe/gCA3Xt04osnHpxxIkmStl9TX3GaBbwSEac18XlVQOYvWcnP//xC/fjyUZW0bZ3r\nlGGSJOWvpv5pdhSwF3A28OcmPrcKQEqJH4x7nlVrqwA49bB9GLx/z4xTSZLUNJq0OKWUngKeAn7X\nlOdV4Xhk8rs88+L7AOzYuS0Xnz4w40SSJDWdnG/VRUTfHPY5b/viqJB9tGItP7x9cv34O+cOpVP7\n1hkmkiSpaTXkGacpEfGt2MyUzxHRMyLuA37RdNFUaH5y51Q+XL4WgKMG9ubTg3bPOJEkSU2rIcXp\nPOA7wNMRsfeGjXVXmV4BugLelylRE1+ZxwPPvg1Ax3at+PaIIRknkiSp6eVcnFJKdwEHAx8A0+uu\nPt0L/D/gauCIlNKbzRNT+Wz12iquGTupfnzxGYPYqWv7DBNJktQ8GvRweEppIfD5iLgNuB5YCVSm\nlF5qjnAqDL+670XmfrACgEH79eCUQ/fexhGSJBWmBs3jFBHdImIccCpwHbAQGB8Rg5ojnPLfK7MW\nM/7RVwFoXVHG5aMqKSv7l8fgJEkqCg35VN3nqH2WaW9gcErpcuAQ4O/AcxHxfyPCWQ5LSFVVDVeN\nmUhNSgB86aR+7NGzc8apJElqPg254nQX8FPgEymlVwFSSitTShcCnwNGA5O3cryKzNhHZvLGnA8B\n2He3bow6dpszVkiSVNAacoVoaErpxc29kFJ6JCL6ATc1TSzlu/cWLOPX99f+cSiL4MrRlVRUuGa0\nJKm45VyctlSaNnp9GfDF7U6kvFdTk7j61omsq6oBYMQxB9C3z44Zp5IkqfnldIkgIobnesKIaB8R\nBzU+kvLdvf94i6mvLwRgl+4d+crJh2ScSJKklpHrvZVbI+KhiDgzIjpsboeI6BsR1wBvAYObLKHy\nyqKPVnHznVPrx5efN4x2bfxMgCSpNOT6E68vcCFwFTAuIl4H5gJrgG7AAUBH4G7gM87rVLxuGD+Z\nFavXA3DiJ/aisu/OGSeSJKnl5FScUkrrgZuBmyNiCPApYA+gHTCd2ofCn0gpLWmuoMre41Pf44lp\nswHo1qkN3zjT6bskSaWlwfdYUkqTcdqBkrN81TpuGP/Pt/1bZw+ha8c2GSaSJKnlNfjz4xExYiuv\n3bB9cZSvfnrXND5YuhqATx2yK8cO3SPjRJIktbzGTLzzi4g4ftONEXETcN72R1K+mfLaAu7+e+36\nze3bVPDdc4cS4bIqkqTS05jiNJLa9ek+tWFDRPwUOAs4qqmCKT+sWVfFNbdOrB9fdNoAeu2w2Q9W\nSpJU9BpcnFJKfwG+BtwXEYMj4hbgNOCoDUuxqHj8z19e5r2FywHot1d3zjhiv4wTSZKUnUZNwJNS\nGhcRXYF/AIuAI1JKbzZpMmXu9dkfcutDrwBQUV7GlaOHU1bmLTpJUunKqThFxI+28NIiYCrwtQ3P\nvKSULm2aaMpSVXUNV42ZQHVNAuCCEw5ir126ZJxKkqRs5XrFaeAWtr8JdN7o9bTdiZQX/vj4a8x8\nt3Zarj137sL5n3UVHUmScp0A04e+S8icRcv5xT3TAYiAK0dX0rpVecapJEnKXmM+VacillLiurGT\nWLu+GoCzjtqfQ/beKeNUkiTlB4uTPuYvz73DxJnzAejZrT0Xnto/40SSJOUPi5PqLV62mpvumFI/\nvmzkMDq0bZVhIkmS8ovFSfV+9McpLFu1DoDjhu3Bpw7ZNeNEkiTlF4uTAPj7i3N4+Pl3AejSoTWX\nnjUk40SSJOUfi5NYuWY91932fP34krMGs0PnthkmkiQpP1mcxC13v8DCD1cBMLzvzpwwfM+ME0mS\nlJ8sTiVu+luL+NOTrwPQtnU5l503jA2zwEuSpI+zOJWwdeuruXrMBFLdfO8XntKfXbt3zDaUJEl5\nzOJUwn6P7bxsAAATa0lEQVT/4AzembcMgL59duTso/fPOJEkSfnN4lSi3pr7Eb97cAYA5WXBFaMq\nKS/zj4MkSVvjT8oSVF1Tw9VjJlJVXQPA6OP6sl/vbhmnkiQp/1mcStBdT77BS29/AMDuPTvxxc/1\nyziRJEmFweJUYuYvXsnP736hfnzFqEratCrPMJEkSYXD4lRCUkpcd9skVq2tAuC0w/dh0H49M04l\nSVLhsDiVkIeff5d/vDwXgO5d2vGfpw3MOJEkSYXF4lQiPlq+hh/ePrl+/N1zh9KxfesME0mSVHgs\nTiXix3dO5aMVawH49KDeHDmwd8aJJEkqPBanEjBhxjz+8tw7AHRs14pvnzM040SSJBUmi1ORW722\nimvHTqwff/2MQXTv2i7DRJIkFS6LU5H75X3Tmbt4JQCD9+vJKZ/aO+NEkiQVLotTEZsxazG3P/oa\nAG1alXP5qGFERMapJEkqXBanIlVVVcPVYyZQkxIAXz6pH7v37JxxKkmSCpvFqUjd+vArvDHnIwD2\n692NkcccmHEiSZIKn8WpCM2av4zfPPASAGURXDm6kooK32pJkraXP02LTE1N4ppbJ7KuqgaAc489\ngAP32DHjVJIkFQeLU5G555k3mfbGQgB27d6Rr5x0SMaJJEkqHhanIrLoo1XcfOe0+vH3zhtG2zYV\nGSaSJKm4WJyKyPXjJ7NyzXoATvrkXlT23TnjRJIkFReLU5F4fOp7PDltNgA7dGrL188clHEiSZKK\nj8WpCCxbuZbrxz1fP/7WiCF06dAmw0SSJBUni1MRuPmuaSxetgaAww7ZlWMG755xIkmSipPFqcBN\nfnU+9z7zFgAd2lbw3ZEuqyJJUnPJm+IUERdFxDsRsToiJkTE0K3se0RE1GzyVR0RPVoyc9bWrKvi\nmrGT6scXnTaQnt3aZ5hIkqTilhfFKSLOBm4E/gsYCEwHHoqI7ls5LAH7Ar3qvnZOKS1s7qz55DcP\nvMTshcsB6L/3Tpx++L4ZJ5IkqbjlRXECLgF+lVIak1J6FfgqsAq4YBvHLUopLdzw1ewp88hrs5cw\n9uGZALSqKOOK0ZWUlXmLTpKk5pR5cYqIVsBg4LEN21JKCXgU+MTWDgVeiIi5EfFwRHyyeZPmj6rq\nGq4eM5HqmgTABScczJ47d8k4lSRJxS/z4gR0B8qBBZtsX0DtLbjNmQd8BTgdOA2YDTwZEQOaK2Q+\nuf2x15j57hIA9tqlC+d/tm/GiSRJKg0FuR5HSul14PWNNk2IiL2pveV3fjapWsachcv55b3TAYiA\nK0cPp1VFecapJEkqDflQnD4AqoGem2zvCcxvwHkmAYdua6dLLrmELl0+fltrxIgRjBgxogH/qGyk\nlLhm7CTWrq8G4Kyj9qffXlt7fl6SpNI2fvx4xo8f/7FtS5cubfT5ovZxomxFxARgYkrp63XjAN4D\nbk4p3ZDjOR4GlqWUztjC64OAKVOmTGHQoMJcjuT+Z9/iv38/AYBeO7Tn9u9/jg5tW2WcSpKkwjJ1\n6lQGDx4MMDilNLUhx+bDFSeAHwG/j4gp1F45ugRoD/weICKuBXZJKZ1fN/468A4wA2gLfBk4Cji2\nxZO3kMXLVvPjO/753l42cpilSZKkFpYXxSmldEfdnE3/Te0tuheA41JKi+p26QX03uiQ1tTO+7QL\ntdMWvAgcnVJ6uuVSt6wbb5/CslXrAPhsZR8O7bdrxokkSSo9eVGcAFJKtwC3bOG1L2wyvgHI6RZe\nMXh6+hwemfwuAF06tOHSswZnnEiSpNKUD9MRaCtWrF7PD8Y9Xz++9OzBdOvUNsNEkiSVLotTnvv5\n3dNY+OEqAIYftDPHV/bJNpAkSSXM4pTHpr+5kDuffAOAtq3L+d7IYdR+4FCSJGXB4pSn1q2v5qox\nE+vHF546gF26d8wwkSRJsjjlqd89OINZ85cBcFCfHTn70/tlnEiSJFmc8tCb73/E7x+cAUB5WXDl\n6ErKy3yrJEnKmj+N80x1TQ1Xj5lAVXUNAOcffxD77NYt41SSJAksTnnnziff4OV3FgOwR8/OXHDC\nwRknkiRJG1ic8si8xSv4+d0v1I+vGF1Jm1blGSaSJEkbszjliZQS1932PKvXVgFw+hH7MnDfHhmn\nkiRJG7M45YmHJs3i2ZfnArBT13b8x+cHZJxIkiRtyuKUBz5avoYb/zilfvzdc4fSsX3rDBNJkqTN\nsTjlgZv+NJWPVqwF4JjBu3PEgN4ZJ5IkSZtjccrYczPm8tcJ7wDQqX1rvnnOkIwTSZKkLbE4ZWjV\nmvVcO3ZS/fgbZwyie5d2GSaSJElbY3HK0K/ue5F5i1cCMOSAnpx06F4ZJ5IkSVtjccrIjHc+4PbH\nXgOgTatyLj+vkojIOJUkSdoai1MG1ldVc9WYidSkBMC/n9SP3j06ZZxKkiRti8UpA7c+NJM33/8I\ngP17d+PcYw/MOJEkScqFxamFzZq3lN/85SUAysuCK0cPp6Lct0GSpELgT+wWVFOTuGbsRNZX1QAw\n8tgDOWCPHTJOJUmScmVxakH3/P1Npr2xCIDddurIlz/XL+NEkiSpISxOLWThh6u4+a5p9ePLR1XS\ntk1FhokkSVJDWZxaQEqJ68c9z8o16wE4+dC9GXpAr4xTSZKkhrI4tYDHp87mqelzANihc1suPmNg\nxokkSVJjWJya2bKVa7l+/PP142+fM4QuHdpkmEiSJDWWxamZ3XzXNJYsWwPA4f134+jBu2ecSJIk\nNZbFqRk9/+p87n3mLQA6tG3Fd88d6rIqkiQVMItTM1mzroprbp1YP/7P0wfQo1v7DBNJkqTtZXFq\nJr9+4CXmLFoBwIB9duLzh+2bcSJJkrS9LE7N4NV3l3DbwzMBaFVRxhWjKikr8xadJEmFzuLUxKqq\na7hqzASqaxIAXzzxYPrs3CXjVJIkqSlYnJrY+Edf5bXZHwKw9y5dGH1c34wTSZKkpmJxakKzFy7n\nV/e9CEAEXDl6OK0qyjNOJUmSmorFqYmklLhm7ETWrq8G4JxPH8DBe3XPOJUkSWpKFqcmcv+zbzP5\n1QUA7LxjB756yiEZJ5IkSU3N4tQEPli6mp/8aWr9+LKRw2jftlWGiSRJUnOwODWBG/84mWWr1gFw\nfGUfPnnwLhknkiRJzcHitJ2eemEOj05+D4CuHdtw6VmDM04kSZKai8VpO6xYtY4fjJtUP7707MF0\n7dQ2w0SSJKk5WZy2w8/ufoFFH60G4JMH78Jnh/XJNpAkSWpWFqdGeuGNhdz11BsAtGtTwWUjhxLh\nsiqSJBUzi1MjrF1fzdW3Tqwff+3U/uy8Y8cME0mSpJZgcWqE3/31ZWbNXwbAwXvuyJlH7ZdxIkmS\n1BIsTg305pwP+f2DMwAoLwuuGD2c8jL/M0qSVAr8id8A1TU1XH3rRKprEgD/dvxB7LNr14xTSZKk\nlmJxaoA7nnidl99ZDECfXp254ISDM04kSZJaksUpR3M/WMEv7pleP75iVCWtW5VnmEiSJLU0i1MO\nUkpce9skVq+tAuCMI/dlwL49Mk4lSZJamsUpB3+bNIsJM+YB0KNrOy76/MCME0mSpCxYnLbhw+Vr\nuPH2KfXj75w7jI7tWmWYSJIkZcXitA033TGFpSvXAnDMkN05YsBuGSeSJElZsThtxbMvz+XBibMA\n6Ny+Nd86Z0i2gSRJUqYsTluwas16rh37z2VVvn7mIHbs3C7DRJIkKWsWpy34xb3Tmb9kFQBDD+jF\nSZ/cK+NEkiQpaxanzXj57Q/44+OvAdCmVTmXnzeMiMg4lSRJyprFaRPrq6q5aswEUu2qKnzllEPY\nrUenbENJkqS8YHHaxJiHXuGtuUsBOGD3HRhx9AEZJ5IkSfnC4rSRWfOW8j9/eRmA8rLgytGVVJT7\nn0iSJNWyFdSpqUlcfetE1lfVADDyMwey/+47ZJxKkiTlE4tTnT8//QYvvLkIgN126siXP9cv40SS\nJCnfWJyABR+u4md/nlY/vmJUJW1bV2SYSJIk5aOSL04pJX5w2yRWrqkC4JRP7c2QA3plnEqSJOWj\nki9Oj015j7+/+D4AO3Zuy8WnD8w4kSRJylclXZyWrlzLDeMn14+/M2IonTu0yTCRJEnKZyVdnG6+\ncxpLlq8B4MgBu3HUoN4ZJ5IkSfmsZIvTpJnzue8fbwHQoW0rvj1iqMuqSJKkrSrJ4rRmbRXX3Dqx\nfnzx6QPp0a19hokkSVIhKMni9P/uf5H3P1gBwMB9e3DqYftknEiSJBWCkpus6J15S7ntkXkAtK4o\n4/JRlZSVeYtOkiRtW95ccYqIiyLinYhYHRETImLoNvY/MiKmRMSaiHg9Is7P5Z/zmwdeoiYlAL70\nuX706dW5CdJre40fPz7rCNoC35v85vuTv3xvilNeFKeIOBu4EfgvYCAwHXgoIrpvYf8+wAPAY0B/\n4CfAbyLi2G39s95dsAyAfXbtyqjP9G2C9GoKfoPJX743+c33J3/53hSnvChOwCXAr1JKY1JKrwJf\nBVYBF2xh/wuBt1NK30kpvZZS+jlwZ915tqksgitHV1JRkS//+pIkqRBk3hwiohUwmNqrRwCklBLw\nKPCJLRw2vO71jT20lf0/5pyj9+egPTd7MUuSJGmLMi9OQHegHFiwyfYFwJYWjeu1hf07R8RWp/7e\nqUs7vnpK/8bklCRJJa6UPlXXFuCoA9owc8aLWWfRJpYuXcrUqVOzjqHN8L3Jb74/+cv3Jn/NnDlz\nw/9t29BjI9V9wiwrdbfqVgGnp5Tu22j774EuKaXPb+aYp4ApKaVLN9r2b8BNKaVuW/jnnAvc1rTp\nJUlSARuZUhrXkAMyv+KUUlofEVOAo4H7AKJ27ZOjgZu3cNhzwPGbbPtM3fYteQgYCcwC1mxHZEmS\nVNjaAn2o7QYNkvkVJ4CIOAv4PbWfpptE7afjzgAOSCktiohrgV1SSufX7d8HeAm4BfgttSXrx8AJ\nKaVNHxqXJElqEplfcQJIKd1RN2fTfwM9gReA41JKi+p26QX03mj/WRFxInATcDEwB/iipUmSJDWn\nvLjiJEmSVAjyYToCSZKkglASxamh6+CpZUTEYRFxX0S8HxE1EXFy1plUKyK+FxGTImJZRCyIiLsj\nYr+scwki4qsRMT0iltZ9PRsRn806l/5VRFxW973tR1lnEUTEf9W9Hxt/vdLQ8xR9cWroOnhqUR2o\nfZ7ta4D3jPPLYcBPgUrgGKAV8HBEtMs0lQBmA98FBlG76sLjwL0RcWCmqfQxdb+g/zu1P3OUP16m\n9lnqXnVfn2roCYr+GaeImABMTCl9vW4c1H7juTmldH2m4VQvImqAUzeey0v5o+4XjYXA4SmlZ7LO\no4+LiMXAt1JKv8s6iyAiOgJTqF1X9X8B0zaed1DZiIj/Ak5JKQ3anvMU9RWnRq6DJ+lfdaX2quCS\nrIPonyKiLCLOAdqz9Xns1LJ+DtyfUno86yD6F/vWPR7yVkSMjYje2z7k4/JiOoJmtLV18PZv+ThS\n4am7Svtj4JmUUoOfB1DTi4iDqS1KbYHlwOdTSq9mm0oAdUV2ADAk6yz6FxOAfwNeA3YGvg88HREH\np5RW5nqSYi9OkrbfLUBf4NCsg6jeq0B/oAu1kwWPiYjDLU/ZiojdqP0l45iU0vqs8+jjUkobzxL+\nckRMAt4FzgJyvs1d7MXpA6Ca2gfBNtYTmN/ycaTCEhE/A04ADkspzcs6j2qllKqAt+uG0yJiGPB1\nap+pUXYGAzsBU+uu1ELtXY/DI+I/gDap2B8sLiAppaUR8TqwT0OOK+pnnOoa/4Z18ICPrYP3bFa5\npEJQV5pOAY5KKb2XdR5tVRnQJusQ4lGgH7W36vrXfU0GxgL9LU35pe4h/n2ABv1SWOxXnAB+BPy+\nbiHhDevgtad2bTxlKCI6UPuHdsNvZntFRH9gSUppdnbJFBG3ACOAk4GVEbHhqu3SlJKLZGcoIq4B\nHgTeAzpRu3j5EdQudK4M1T0n87HnACNiJbA4pTQzm1TaICJuAO6n9vbcrsD/AdYD4xtynqIvTjms\ng6fsDAGeoPbTWona+bYA/gBckFUoAbULbifgyU22fwEY0+JptLEe1P4d2RlYCrwIfMZPcOUtrzLl\nj92AccCOwCLgGWB4SmlxQ05S9PM4SZIkNZWifsZJkiSpKVmcJEmScmRxkiRJypHFSZIkKUcWJ0mS\npBxZnCRJknJkcZIkScqRxUmSJClHFidJkqQcWZwkSZJyZHGSVNQioiwi/hERd22yvXNEvBcR/zer\nbJIKj2vVSSp6EbEvMA34ckppfN22MUA/YGhKqSrLfJIKh8VJUkmIiP8Evg/0BYYDfwSGpJRezjKX\npMJicZJUMiLiMaCG2itNP0kpXZtxJEkFxuIkqWRExP7ATOBFYFBKqSbjSJIKjA+HSyolXwRWAnsC\nu2WcRVIB8oqTpJIQEZ8EngA+A1xJ7fe/Y7JNJanQeMVJUtGLiHbA74BbUkpPAV8ChkbEV7JNJqnQ\nWJwklYLr6v73ewAppXeBbwM3RMTumaWSVHC8VSepqEXE4cCjwBEppec2ee1BoCKldGwm4SQVHIuT\nJElSjrxVJ0mSlCOLkyRJUo4sTpIkSTmyOEmSJOXI4iRJkpQji5MkSVKOLE6SJEk5sjhJkiTlyOIk\nSZKUI4uTJElSjixOkiRJObI4SZIk5ej/A3USkIXiux9KAAAAAElFTkSuQmCC\n", "text/plain": [ "" ] }, "metadata": {}, "output_type": "display_data" }, { "data": { "image/png": "iVBORw0KGgoAAAANSUhEUgAAAk4AAAGFCAYAAAALnnwfAAAABHNCSVQICAgIfAhkiAAAAAlwSFlz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\nAElFTkSuQmCC\n", "text/plain": [ "" ] }, "metadata": {}, "output_type": "display_data" } ], "source": [ "kb = GPy.kern.Brownian(input_dim=1)\n", "inputs = np.array([2., 4.])\n", "for x in inputs:\n", " kb.plot(x,plot_limits=[0,5])\n", "plt.legend(inputs)\n", "plt.ylim(-0.1,5.1)" ] }, { "cell_type": "markdown", "metadata": {}, "source": [ "### Exercise 2\n", "\n", "Plot some other covariance functions. As we will see, it is very hard to intuit from the shape of these what the resulting GP sample paths will look like. \n", "\n" ] }, { "cell_type": "markdown", "metadata": {}, "source": [ "## Computing the covariance/Gram matrix given the input data, $\\mathbf{X}$" ] }, { "cell_type": "markdown", "metadata": {}, "source": [ "Let $\\mathbf{X}$ be a $n$ × $d$ numpy array. Given a kernel $k$, the covariance matrix associated to\n", "$\\mathbf{X}$ is obtained with `C = k.K(X,X)` . The positive semi-definiteness of $k$ ensures that `C`\n", "is a positive semi-definite (psd) matrix regardless of the initial points $\\mathbf{X}$. This can be\n", "checked numerically by looking at the eigenvalues (they should all be positive if C is positive definite):" ] }, { "cell_type": "code", "execution_count": 9, "metadata": { "collapsed": false }, "outputs": [ { "data": { "text/plain": [ "" ] }, "execution_count": 9, "metadata": {}, "output_type": "execute_result" }, { "data": { "image/png": "iVBORw0KGgoAAAANSUhEUgAAAgUAAAFyCAYAAABlU6npAAAABHNCSVQICAgIfAhkiAAAAAlwSFlz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\n", "text/plain": [ "" ] }, "metadata": {}, "output_type": "display_data" } ], "source": [ "k = GPy.kern.Matern52(input_dim=2)\n", "X = np.random.rand(50,2) # 50*2 matrix of iid standard Gaussians\n", "C = k.K(X,X)\n", "eigvals = np.linalg.eigvals(C) # Computes the eigenvalues of a matrix\n", "plt.bar(np.arange(len(eigvals)), eigvals)\n", "plt.title('Eigenvalues of the Matern 5/2 Covariance')" ] }, { "cell_type": "markdown", "metadata": {}, "source": [ "## 2. Prior Simulation" ] }, { "cell_type": "markdown", "metadata": {}, "source": [ "Gaussian processes can be used as infinite dimensional models of function. They are defined by a covariance *function* and a mean *function*. \n", "\n", "When we compute the covariance matrix using `kern.K(X, X)` we are computing a covariance *matrix* between the values of the function, $f$, that correspond to the input locations in the matrix `X`. If we want to have a look at the type of functions that arise from a particular Gaussian process, we can never generate all possible values, but we can generate random samples from a Gaussian *distribution* based on a covariance matrix associated with a particular matrix of input locations `X`. If these locations are chosen appropriately then they give us a good idea of the underlying function. For example, for a one dimensional function, if we choose `X` to be uniformly spaced across part of the real line, and the spacing is small enough, we'll get an idea of the underlying function. We will now use this trick to draw sample paths from a Gaussian process. " ] }, { "cell_type": "code", "execution_count": 10, "metadata": { "collapsed": false }, "outputs": [ { "name": "stderr", "output_type": "stream", "text": [ " /Users/pmzrdw/anaconda/lib/python3.5/site-packages/ipykernel/__main__.py:10: RuntimeWarning:covariance is not positive-semidefinite.\n" ] }, { "data": { "image/png": "iVBORw0KGgoAAAANSUhEUgAAAgoAAAFkCAYAAABB1xPiAAAABHNCSVQICAgIfAhkiAAAAAlwSFlz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\n", "text/plain": [ "" ] }, "metadata": {}, "output_type": "display_data" } ], "source": [ "k = GPy.kern.RBF(input_dim=1,lengthscale=0.2)\n", "\n", "X = np.linspace(0.,1.,500) # define X to be 500 points evenly spaced over [0,1]\n", "X = X[:,None] # reshape X to make it n*p \n", "\n", "mu = np.zeros((500)) # vector of the means --- we could use a mean function, but here it is just zero.\n", "C = k.K(X,X) # compute the covariance matrix associated with inputs X\n", "\n", "# Generate 20 separate samples paths from a Gaussian with mean mu and covariance C\n", "Z = np.random.multivariate_normal(mu,C,20)\n", "\n", "plt.figure() # open a new plotting window\n", "for i in range(20):\n", " plt.plot(X[:],Z[i,:])" ] }, { "cell_type": "markdown", "metadata": {}, "source": [ "Our choice of `X` means that the points are close enough together to look like functions. We can see the structure of the covariance matrix we are plotting from if we visualize C." ] }, { "cell_type": "code", "execution_count": 11, "metadata": { "collapsed": false }, "outputs": [ { "data": { "text/plain": [ "" ] }, "execution_count": 11, "metadata": {}, "output_type": "execute_result" }, { "data": { "image/png": "iVBORw0KGgoAAAANSUhEUgAAAXcAAAFkCAYAAADbgnvLAAAABHNCSVQICAgIfAhkiAAAAAlwSFlz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\n", "text/plain": [ "" ] }, "metadata": {}, "output_type": "display_data" } ], "source": [ "plt.matshow(C)" ] }, { "cell_type": "markdown", "metadata": {}, "source": [ "### Exercise 3: \n", "Try a range of different covariance functions and hyper-parameter values and plot the corresponding sample paths for each using the same approach given above. Compare these to the shapes of the covariance functions you found in Exercise 2. Can you see any relationship?" ] }, { "cell_type": "code", "execution_count": 12, "metadata": { "collapsed": false }, "outputs": [], "source": [ "# Try plotting sample paths here" ] }, { "cell_type": "markdown", "metadata": {}, "source": [ "### Exercise 4:" ] }, { "cell_type": "markdown", "metadata": {}, "source": [ "The sample paths from a GP inherit their properties (such as continuity, differentiability, smoothness etc) from the particular covariance function used.\n", "\n", "Can you tell the covariance structures \n", "that have been used for generating the\n", "sample paths shown in the figure below?\n", "
\n", "
\n", "\"Figure \n", "\"Figure\n", "\"Figure \n", "\"Figure\n", "\"Figure \n", "\"Figure\n", "
\n" ] }, { "cell_type": "raw", "metadata": {}, "source": [ "# Exercise 4 answer" ] }, { "cell_type": "markdown", "metadata": {}, "source": [ "## 3. A Gaussian Process Regression Model" ] }, { "cell_type": "markdown", "metadata": {}, "source": [ "We will now combine the Gaussian process prior with some data to form a GP regression model with GPy. We will generate data from the function $f ( x ) = 0.3\\cos(1.3 x ) + \\sin(0.3x )$ over $[0, 10]$, adding some noise to give $y(x) = f(x) + \\epsilon$, with the noise being Gaussian distributed, $\\epsilon \\sim \\mathcal{N}(0, 0.01)$. " ] }, { "cell_type": "code", "execution_count": 13, "metadata": { "collapsed": false }, "outputs": [ { "data": { "text/plain": [ "" ] }, "execution_count": 13, "metadata": {}, "output_type": "execute_result" }, { "data": { "image/png": "iVBORw0KGgoAAAANSUhEUgAAAgkAAAFkCAYAAACq4KjhAAAABHNCSVQICAgIfAhkiAAAAAlwSFlz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\nrkJggg==\n", "text/plain": [ "" ] }, "metadata": {}, "output_type": "display_data" } ], "source": [ "X = np.random.uniform(0, 10, (200, 1))\n", "f = np.sin(.3*X) + .3*np.cos(1.3*X)\n", "Y = f+np.random.normal(0, .1, f.shape)\n", "plt.scatter(X, Y)\n" ] }, { "cell_type": "markdown", "metadata": {}, "source": [ "A GP regression model based on an exponentiated quadratic covariance function can be defined by first defining a covariance function, " ] }, { "cell_type": "code", "execution_count": 14, "metadata": { "collapsed": false }, "outputs": [], "source": [ "k = GPy.kern.RBF(input_dim=1, variance=1., lengthscale=1.)" ] }, { "cell_type": "markdown", "metadata": {}, "source": [ "And then combining it with the data to form a Gaussian process model," ] }, { "cell_type": "code", "execution_count": 15, "metadata": { "collapsed": false }, "outputs": [ { "name": "stdout", "output_type": "stream", "text": [ "\n", "Name : GP regression\n", "Objective : 199.15646490667305\n", "Number of Parameters : 3\n", "Number of Optimization Parameters : 3\n", "Updates : True\n", "Parameters:\n", " \u001b[1mGP_regression. \u001b[0;0m | value | constraints | priors\n", " \u001b[1mrbf.variance \u001b[0;0m | 1.0 | +ve | \n", " \u001b[1mrbf.lengthscale \u001b[0;0m | 1.0 | +ve | \n", " \u001b[1mGaussian_noise.variance\u001b[0;0m | 1.0 | +ve | \n" ] } ], "source": [ "m = GPy.models.GPRegression(X,Y,k)\n", "print(m)" ] }, { "cell_type": "markdown", "metadata": {}, "source": [ "Just as for the covariance function object, we can find out about the model using the command `print m`. " ] }, { "cell_type": "code", "execution_count": 16, "metadata": { "collapsed": false }, "outputs": [ { "name": "stdout", "output_type": "stream", "text": [ "\n", "Name : GP regression\n", "Objective : 199.15646490667305\n", "Number of Parameters : 3\n", "Number of Optimization Parameters : 3\n", "Updates : True\n", "Parameters:\n", " \u001b[1mGP_regression. \u001b[0;0m | value | constraints | priors\n", " \u001b[1mrbf.variance \u001b[0;0m | 1.0 | +ve | \n", " \u001b[1mrbf.lengthscale \u001b[0;0m | 1.0 | +ve | \n", " \u001b[1mGaussian_noise.variance\u001b[0;0m | 1.0 | +ve | \n" ] } ], "source": [ "print(m)" ] }, { "cell_type": "markdown", "metadata": {}, "source": [ "Note that by default the model includes some observation noise\n", "with variance 1. We can see the posterior mean prediction and visualize the marginal posterior variances using `m.plot()`." ] }, { "cell_type": "code", "execution_count": 17, "metadata": { "collapsed": false }, "outputs": [ { "data": { "text/plain": [ "" ] }, "execution_count": 17, "metadata": {}, "output_type": "execute_result" }, { "name": "stderr", "output_type": "stream", "text": [ " /Users/pmzrdw/anaconda/lib/python3.5/site-packages/matplotlib/figure.py:1742: UserWarning:This figure includes Axes that are not compatible with tight_layout, so its results might be incorrect.\n" ] }, { "data": { "image/png": "iVBORw0KGgoAAAANSUhEUgAAAk4AAAGGCAYAAACNCg6xAAAABHNCSVQICAgIfAhkiAAAAAlwSFlz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\n", "text/plain": [ "" ] }, "metadata": {}, "output_type": "display_data" } ], "source": [ "m.plot()" ] }, { "cell_type": "markdown", "metadata": {}, "source": [ "The actual predictions of the model for a set of points `Xstar`\n", "(an $m \\times p$ array) can be computed using \n", "\n", "`Ystar, Vstar = m.predict(Xstar)`" ] }, { "cell_type": "code", "execution_count": 18, "metadata": { "collapsed": false }, "outputs": [], "source": [ "Xp = np.linspace(-2,12)[:,None]\n", "Ystar, Vstar = m.predict(Xp, full_cov=True)\n", "up95, lo95 = m.predict_quantiles(Xp, quantiles=(2.5,97.5))\n" ] }, { "cell_type": "markdown", "metadata": {}, "source": [ "Note that the inputs to GPy need to be 2-d arrays." ] }, { "cell_type": "code", "execution_count": 19, "metadata": { "collapsed": true }, "outputs": [], "source": [ "Xp = np.linspace(-2,12)[:,np.newaxis]" ] }, { "cell_type": "markdown", "metadata": {}, "source": [ "### Exercise 5" ] }, { "cell_type": "markdown", "metadata": {}, "source": [ "a) What do you think about this first fit? Does the prior given by the GP seem to be\n", "adapted?" ] }, { "cell_type": "raw", "metadata": {}, "source": [ "# Exercise 4 a) answer" ] }, { "cell_type": "markdown", "metadata": {}, "source": [ "b) The parameters of the models can be modified using a regular expression matching the parameters names (for example `m['.*noise'] = 0.001` ), or by `m.Gaussian_noise.variance=0.001`. Change the values of the parameters to obtain a better fit, replotting your fitted model." ] }, { "cell_type": "code", "execution_count": 20, "metadata": { "collapsed": false }, "outputs": [], "source": [ "# Exercise 4 b) answer" ] }, { "cell_type": "markdown", "metadata": {}, "source": [ "c) As in Section 2, random sample paths from the conditional GP can be obtained using\n", "`np.random.multivariate_normal(mu[:,0],C)` where the mean vector and covariance\n", "matrix `mu` and `C` are obtained through the predict function `mu, C = m.predict(Xp,full_cov=True)`. Obtain 10 samples from the posterior sample and plot them alongside the data below." ] }, { "cell_type": "code", "execution_count": 21, "metadata": { "collapsed": false }, "outputs": [], "source": [ "# Exercise 4 c) answer" ] }, { "cell_type": "markdown", "metadata": {}, "source": [ "Note that the `GPy.models` commands are wrappers for a collection of commands. `GPRegression` makes two default choices here:\n", "\n", "1. Specifies a Gaussian likelihood, i.e., we assume that $y=f(x) + e$ where $f$ is the GP, and $e\\sim N(0, \\sigma^2)$ is the Gaussian noise term (referred to as the likelihood in GPy), sometimes called a **nugget** in spatial statistics.\n", "2. Specifies an inference scheme, i.e., a way of calculating the posterior distribution of $f$ given the training data. Because we are using a Gaussian likelihood here, the posterior distribution of $f$ is also a Gaussian process, and so inference can be done exactly.\n", "\n", "We could equivalently set up the model using the following:\n" ] }, { "cell_type": "code", "execution_count": 22, "metadata": { "collapsed": false }, "outputs": [ { "name": "stdout", "output_type": "stream", "text": [ "\n", "Name : gp\n", "Objective : 199.15646490667305\n", "Number of Parameters : 3\n", "Number of Optimization Parameters : 3\n", "Updates : True\n", "Parameters:\n", " \u001b[1mgp. \u001b[0;0m | value | constraints | priors\n", " \u001b[1mrbf.variance \u001b[0;0m | 1.0 | +ve | \n", " \u001b[1mrbf.lengthscale \u001b[0;0m | 1.0 | +ve | \n", " \u001b[1mGaussian_noise.variance\u001b[0;0m | 1.0 | +ve | \n" ] } ], "source": [ "gauss = GPy.likelihoods.Gaussian(variance=1.0)\n", "exact = GPy.inference.latent_function_inference.ExactGaussianInference()\n", "m1 = GPy.core.GP(X=X, Y=Y, kernel=k, likelihood=gauss, inference_method=exact)\n", "print(m1)" ] }, { "cell_type": "markdown", "metadata": {}, "source": [ "### Covariance Function Parameter Estimation" ] }, { "cell_type": "markdown", "metadata": {}, "source": [ "As we have seen during the lectures, the parameters values can be estimated by maximizing the likelihood of the observations. Since we don’t want one of the variance to become negative during the optimization, we can constrain all parameters to be positive before running the optimisation." ] }, { "cell_type": "code", "execution_count": 23, "metadata": { "collapsed": false }, "outputs": [ { "name": "stdout", "output_type": "stream", "text": [ "WARNING: reconstraining parameters GP_regression\n" ] } ], "source": [ "m.constrain_positive()" ] }, { "cell_type": "markdown", "metadata": {}, "source": [ "The warnings are because the parameters are already constrained by default, the software is warning us that they are being reconstrained." ] }, { "cell_type": "markdown", "metadata": {}, "source": [ "Now we can optimize the model using the `m.optimize()` method. A number of optimizers are available, such as 'scg', 'lbfgs', 'org-bfgs', 'fmin_tnc', as well as stochastic optimizers if a dependency on a package 'climin' is satisfied, such as 'adadelta' and 'rprop'." ] }, { "cell_type": "code", "execution_count": 24, "metadata": { "collapsed": false }, "outputs": [ { "name": "stdout", "output_type": "stream", "text": [ "\n", "Name : GP regression\n", "Objective : -151.23345681629752\n", "Number of Parameters : 1\n", "Number of Optimization Parameters : 1\n", "Updates : True\n", "Parameters:\n", " \u001b[1mGP_regression. \u001b[0;0m | value | constraints | priors\n", " \u001b[1mGaussian_noise.variance\u001b[0;0m | 0.00895683789669 | +ve | \n" ] }, { "name": "stderr", "output_type": "stream", "text": [ " /Users/pmzrdw/anaconda/lib/python3.5/site-packages/matplotlib/figure.py:1742: UserWarning:This figure includes Axes that are not compatible with tight_layout, so its results might be incorrect.\n" ] }, { "data": { "image/png": "iVBORw0KGgoAAAANSUhEUgAAAk4AAAGGCAYAAACNCg6xAAAABHNCSVQICAgIfAhkiAAAAAlwSFlz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\nSZIkqSWDkyRJUkv/AxlP6D5DRPfXAAAAAElFTkSuQmCC\n", "text/plain": [ "" ] }, "metadata": {}, "output_type": "display_data" } ], "source": [ "m.optimize(messages=1, ipython_notebook=True) # Messages indicates you wish to see the progress of the optimizer, needs ipywidgets to be installed\n", "m.plot()\n", "print(m)" ] }, { "cell_type": "markdown", "metadata": {}, "source": [ "The parameters obtained after optimisation can be compared with the values you selected by hand. As previously, you can modify the kernel used for building the model to investigate its influence on the model." ] }, { "cell_type": "markdown", "metadata": {}, "source": [ "Note what has happened outside the range of the data: the GP prediction returns to its prior mean value of 0. We could fix this by adding a prior mean." ] }, { "cell_type": "markdown", "metadata": { "collapsed": true }, "source": [ "## Problems with hyper-parameter optimization" ] }, { "cell_type": "markdown", "metadata": {}, "source": [ "Unfortunately, optimization of GP hyper-parameters is not always without problems. The likelihood surface that is being maximized is often flat and multi-modal, and thus the optimizer can sometimes fail to converge, or gets stuck in local-maxima. You can see from the optimization output above, that the gradients at the optima are very small ($10^{-18}$), but not necessarily zero. This kind of problem is due to the use of numerical optimization. When using GPs this can type of problem is common, and so we must always sanity check our answers, and ideally run some diagnostic tests on the fitted model.\n", "\n", "As an example, consider\n" ] }, { "cell_type": "code", "execution_count": 25, "metadata": { "collapsed": false }, "outputs": [], "source": [ "np.random.seed(16)\n", "X = np.linspace(0.05,0.95,10)[:,None]\n", "Y = -np.cos(np.pi*X) + np.sin(4*np.pi*X) + np.random.normal(loc=0.0, scale=0.1, size=(10,1)) " ] }, { "cell_type": "markdown", "metadata": {}, "source": [ "If we now fit a GP and optimize the hyper parameters, you may find some problems." ] }, { "cell_type": "code", "execution_count": 26, "metadata": { "collapsed": false }, "outputs": [ { "data": { "text/plain": [ "" ] }, "execution_count": 26, "metadata": {}, "output_type": "execute_result" }, { "name": "stderr", "output_type": "stream", "text": [ " /Users/pmzrdw/anaconda/lib/python3.5/site-packages/matplotlib/figure.py:1742: UserWarning:This figure includes Axes that are not compatible with tight_layout, so its results might be incorrect.\n" ] }, { "data": { "image/png": "iVBORw0KGgoAAAANSUhEUgAAAk4AAAGGCAYAAACNCg6xAAAABHNCSVQICAgIfAhkiAAAAAlwSFlz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\nx3qA8k0wFwAAAABJRU5ErkJggg==\n", "text/plain": [ "" ] }, "metadata": {}, "output_type": "display_data" } ], "source": [ "m = GPy.models.GPRegression(X,Y) # this will automatically use a RBF covariance function\n", "m.optimize(messages=1)\n", "m.plot()" ] }, { "cell_type": "markdown", "metadata": {}, "source": [ "### Exercise 5:\n", "What may have happened here?\n" ] }, { "cell_type": "code", "execution_count": null, "metadata": { "collapsed": true }, "outputs": [], "source": [] }, { "cell_type": "markdown", "metadata": {}, "source": [ "One way to fix this is to change the parameter settings so that the optimizer starts in a more sensible location as described above. \n", "\n", "Another option is to run the optimizer multiple times from multiple different starting points, and pick the best possible (I'd recommend this as good practice). This can be done in GPy as follows:" ] }, { "cell_type": "code", "execution_count": 27, "metadata": { "collapsed": false }, "outputs": [ { "name": "stdout", "output_type": "stream", "text": [ "Optimization restart 1/10, f = 11.96887713535652\n", "Optimization restart 2/10, f = 7.969923622373031\n", "Optimization restart 3/10, f = 7.969923622373408\n", "Optimization restart 4/10, f = 7.969923622374876\n", "Optimization restart 5/10, f = 11.968877507245928\n", "Optimization restart 6/10, f = 7.969923622373341\n", "Optimization restart 7/10, f = 7.969923622373198\n", "Optimization restart 8/10, f = 7.969923622377622\n", "Optimization restart 9/10, f = 7.96992362237698\n", "Optimization restart 10/10, f = 7.969923622373035\n" ] }, { "data": { "text/plain": [ "" ] }, "execution_count": 27, "metadata": {}, "output_type": "execute_result" }, { "name": "stderr", "output_type": "stream", "text": [ " /Users/pmzrdw/anaconda/lib/python3.5/site-packages/matplotlib/figure.py:1742: UserWarning:This figure includes Axes that are not compatible with tight_layout, so its results might be incorrect.\n" ] }, { "data": { "image/png": "iVBORw0KGgoAAAANSUhEUgAAAk4AAAGGCAYAAACNCg6xAAAABHNCSVQICAgIfAhkiAAAAAlwSFlz\nAAAPYQAAD2EBqD+naQAAIABJREFUeJzs3Xl8VNX5+PHPnX3NTFYCYQkERHZZFEUBUcQFcENRqHWj\nivL7arEutFJtpdS1aqtWoZa6gNhWRcWlBRVRQcUKKCggEAKELIQssyWZ/f7+CIzEbAPJZON5v168\nXs6dc+89MyaZZ855znMUVVURQgghhBBN07R1B4QQQgghOgoJnIQQQggh4iSBkxBCCCFEnCRwEkII\nIYSIkwROQgghhBBxksBJCCGEECJOEjgJIYQQQsRJAichhBBCiDhJ4CSEEEIIEScJnIQQQggh4pTQ\nwElRlFsURflWURT34X+fK4pyQSLvKYQQQgiRKEoi96pTFGUyEAF2AQpwPXA3cIqqqtsTdmMhhBBC\niARIaOBU7w0VpQy4S1XVF1r1xkIIIYQQzaRrrRspiqIBpgMW4IvWuq8QQgghREtJeOCkKMpgagIl\nE+AFLlNVdUei7yuEEEII0dISPlWnKIoO6Ak4gCuAm4Bx9QVPiqKkAucDewF/QjsmhBBCCFHDBGQD\nq1RVLWusYVvkOH0A7FZV9dZ6npsJvNKqHRJCCCGEqPEzVVWXN9ag1XKcjqIBjA08txdg2bJlDBgw\noNU61FHccccdPPnkk23djQ5L3r/mk/eweeT9az55D5tH3r/6bd++nWuuuQYOxyGNSWjgpCjKg8B/\ngP2AHfgZMB6Y1MApfoABAwYwYsSIRHatQ3I4HPK+NIO8f80n72HzyPvXfPIeNo+8f01qMk0o0SNO\nGcBLQFfADWwBJqmquibB9xVCCCGEaHEJDZxUVf1FIq8vhBBCCNGaZK86IYQQQog4SeDUgcyYMaOt\nu9ChyfvXfPIeNo+8f80n72HzyPvXfK1ejqAxiqKMADZu3LhRkteEEOIEsX//fkpLS9u6G6KTS0tL\no2fPnvU+t2nTJkaOHAkwUlXVTY1dpy3KEQghhBBATdA0YMAAqqqq2roropOzWCxs3769weApXhI4\nCSGEaDOlpaVUVVVJ/T6RUEfqNJWWlkrgJIQQouOT+n2io5DkcCGEEEKIOEngJIQQQggRJwmchBBC\nCCHiJIGTEEIIIUScJHASQgghhIiTBE5CCCFEgrz00ktoNBo0Gg2ff/55vW169OiBRqPh4osvbuXe\nieMhgZMQQgiRYGazmeXLl9c5/sknn1BQUIDJZGqDXonjIYGTEEIIkWAXXXQRr732GtFotNbx5cuX\nM2rUKDIzM9uoZ+JYSeAkhBBCJJCiKMyYMYOysjI++OCD2PFQKMTrr7/OzJkz+em+saqq8uc//5nB\ngwdjNpvJzMzklltuweVy1Wq3cuVKpkyZQlZWFiaTib59+7Jw4cI6AdrZZ5/N0KFD2b59OxMmTMBq\ntdK9e3cee+yxxL3wTkoCJyGEECLBsrOzOf3003n11Vdjx95//308Hg9XX311nfY333wz8+bNY+zY\nsTz11FPceOONvPLKK1xwwQVEIpFYuxdffBG73c6dd97JU089xahRo7j//vv5zW9+U+t6iqJQXl7O\nhRdeyPDhw3niiScYMGAAv/71r1m1alXiXngnJFuuCCGEEK1g5syZ3HvvvQQCAYxGI8uXL2f8+PF1\npunWrVvHkiVLePXVV7nqqqtixydMmMD555/Pa6+9Fgu2Xn31VYxGY6zNzTffTHJyMs8++ywLFy5E\nr9fHnisqKmLp0qXMnDkTgBtvvJFevXqxZMkSzj///ES+9E5FAichhBAdxrV//A9l7uqE3iPVYebl\n+Re2+HWnT5/O3Llzeffddzn//PN59913eeaZZ+q0e+2113A6nZx77rmUlZXFjg8fPhybzcbHH38c\nC5yODpp8Ph+BQICzzjqLv/3tb+zYsYMhQ4bEnrfZbLGgCUCv13PaaaexZ8+eFn+tnZkETkIIITqM\nMnc1Ja7EBk6JkpaWxsSJE1m+fDmVlZVEo1GuuOKKOu12796Ny+UiIyOjznOKolBSUhJ7vG3bNubP\nn8/HH3+Mx+Op1c7tdtc6t3v37nWul5yczNatW5vzsk44EjgJIYToMFId5g59j5kzZ3LTTTdRVFTE\nhRdeiN1ur9MmGo3SpUsXli9fXidpHCA9PR0At9vNuHHjcDqdLFy4kD59+mAymdi4cSO//vWv6ySI\na7XaevtU3z1EwyRwEkII0WEkYgqtNV122WXMnj2bDRs28K9//aveNjk5OXz00UeMGTOm1lTcT61d\nu5aKigrefvttzjzzzNjx3NzcFu+3+JGsqhNCCCFaidVqZdGiRfz+979n6tSp9baZPn064XCYBQsW\n1HkuEonEpuC0Wi2qqtYaWQoGgzz77LOJ6bwAZMRJCCGESKifToX9/Oc/b7T9uHHjmD17Ng8//DDf\nfPMNkyZNQq/Xs3PnTl5//XWeeuopLr/8csaMGUNycjLXXnstt99+OwDLli1DUZSEvRYhgZMQQgiR\nUPEEMoqi1Gr33HPPMWrUKBYvXsz8+fPR6XRkZ2dz7bXXxqblUlJSeO+997jzzju57777SE5O5uc/\n/znnnHNOveUFGuqHBFrHRmlPSWGKoowANm7cuJERI0a0dXeEEEIk2KZNmxg5ciTyd18kUlM/Z0ee\nB0aqqrqpsWtJjpMQQgghRJwkcBJCCCGEiJMETkIIIYQQcZLASQghhBAiThI4CSGEEELEScoRCCGE\nEOKEsCu/DEtqKVaTHofNiMVkQKc9tjEkCZyEEEIIcUKw2yxYLGb8wTDuEh+RSBStBorLvHFfQwIn\nIYQQQpwwFEXBZNRjMupjx4KR+M+XHCchhBBCnNAM+vjHkSRwEkIIIYSIkwROQgghhBBxksBJCCGE\nECJOEjgJIYQQCfLSSy+h0Whi/8xmM1lZWVxwwQU8/fTT+Hy+47ruF198wQMPPIDH42nhHoumSOAk\nhBBCJJCiKCxcuJBly5axaNEibr/9dhRFYe7cuQwZMoStW7ce8zU///xzFixYgMvlSkCPRWOkHIEQ\nQgiRYBdccAEjRoyIPZ43bx5r165l8uTJXHLJJWzfvh2j0Rj39VRVTUQ3RRxkxEkIIUSns2/fvlqP\nA4EAxcXFbdSb+p199tncd9997Nu3j2XLlgGwdetWbrjhBnJycjCbzXTt2pVZs2ZRXl4eO++BBx7g\nnnvuASA7OxuNRoNWq2X//v0AvPDCC5x77rl06dIFk8nEoEGDWLRoUeu/wE5KAichhBCdyiOPPMLJ\nJ5/Mhx9+CNQETdOmTWPs2LEcOHCgjXtX289//nNUVWX16tUAfPDBB+Tl5XHjjTfyzDPPMGPGDP75\nz38yefLk2DnTpk1jxowZAPzlL39h2bJlLF26lPT0dAAWLVpEdnY28+fP54knnqBnz57MmTOH5557\nrvVfYCckU3VCCCHatXA4zG233catt97K0KFDASguLuaee+7h6aefxuFwxNpGIhE+++wz/H4/U6dO\n5fXXX+e5557jvffew2QysXv3brp3717r+tFolEgkgl7/YyXpQCBwTFNnxysrKwuHw0Fubi4A/+//\n/T9+9atf1WozevRoZs6cyfr16znzzDMZPHgwI0aM4J///CeXXHIJPXv2rNX+008/rdX3OXPmcOGF\nF/LEE09w6623Jvw1dXYy4iSEEKJdW7hwIYsWLeKcc85hy5YtFBcXM2HCBJYuXcovfvGLWm21Wi1v\nvPEGkydPxu/3M2XKlFjQ9M4773D22WfXah+NRpk9ezbTp08nGAwCUF5ezpgxY3jyySdb5fXZbDa8\n3pq90o4OeAKBAGVlZYwePRpVVdm0aVNc1zv6Gh6Ph7KyMsaNG8eePXti9xHHTwInIYQQ7drcuXM5\n9dRTKSsrY9iwYXTt2pUdO3bQo0cPHn744TrtjUYjy5cvr3Xsl7/8JRMnTqzT9rvvvmPp0qW89dZb\nXHXVVRw8eJCJEyeyadMmHn744Vq5RYni8/mw2+0AVFRU8Mtf/pLMzEzMZjPp6en06dMHRVFwu91x\nXW/9+vVMnDgRm82G0+kkPT2d+fPnA8R9DdEwCZyEEEK0a06nk9WrV9OjR49axz/++GNycnLqtA8E\nAsycObPWsb/85S+xnKejDR06lLfeeguj0chbb71FZmYmmzdvJj09nTVr1pCSktKyL+YnCgoKcLvd\n9OvXD4Arr7ySJUuWMGfOHN58800++OADVq1ahaqqRKPRJq+3Z88eJk6cSHl5OU8++STvv/8+H374\nIXfccQdAXNcQjUto4KQoym8URflKURSPoigHFUV5U1GUkxJ5TyGEEJ2P3+8nHA7XOlZZWVmnXSQS\nYdq0abHpuXfffTc2bTd16lTWrl1b55wLLriAF154odaxVatWMWjQoBZ9DfV5+eWXURSF888/H5fL\nxZo1a/jNb37D/fffzyWXXMK5555L796965ynKEq913vnnXcIBoO888473HTTTVxwwQWcc845mEym\nRL+UE0aiR5zGAk8Do4GJgB5YrSiKOcH3FUII0UkcyWkqKirCbreTmpoKEMt5OppWq2Xs2LGxnKbJ\nkyfHcp66d+9O375961y/vLycxx57rNaxBQsWxHKeEmXNmjUsXLiQPn36MHPmTLRaLVB3VOjJJ5+s\nEyhZrVaAOgUw67uG2+3mxRdfbOnun7ASuqpOVdWLjn6sKMr1QAkwEliXyHsLIYToHJYsWRLLafr4\n449JTU1l0qRJ/O9//+Phhx+uk880b948rr76anr16gXU5Dy98cYbVFRUkJmZWatteXk5EydOjE3P\nzZs3j/nz58dynv71r39hMBia1X9VVXn//ffZvn074XCYgwcPsmbNGj744AN69+7NypUrMRgMGAwG\nxo0bx6OPPkowGCQrK4vVq1ezd+/eOgUvR44ciaqq3HvvvVx99dXo9XouvvhiJk2ahF6vZ8qUKcye\nPRuv18vf//53unTp0u7qWHVUrV2OwAmoQOKz7YQQQnQK9957L4FAgOuuuy6W07R69Wr+8Ic/sHDh\nwnrPORI0HWE0GusETVAz3edyuUhPT+fjjz9m0KBBDBo0iEsvvZRDhw4RDAabHTgpisLvfvc7AAwG\nAykpKQwZMoSnnnqK66+/PjZ6BPDqq69y22238eyzz6KqKueffz7/+c9/6NatW61Rp1GjRsVWG65a\ntYpoNEpeXh4nnXQSb7zxBr/97W+5++67yczMZM6cOaSmpjJr1qxmvQ5RQ2mtsu1Kzf/xdwC7qqrj\nG2gzAti4cePGWqXphRBCdE6bNm1i5MiRtOXf/X379uHz+WrlNK1bt45hw4bFVruJju3Iz9l7H65j\nyLDhdZ7f+u1mJk88C2CkqqqN1n1ozRGnZ4GBwJmteE8hhBCiUT8dnQI466yz2qAnoiNolcBJUZRn\ngIuAsaqqFjXV/o477qhVCRZgxowZsRLzQgghhBDH4+0V/+btFa/VOubxxF/fKuGB0+Gg6RJgvKqq\n++M558knn5SpOiGEEEK0uEsun84ll0+vdeyoqbomJTRwUhTlWWAGcDFQqShKl8NPuVVV9Sfy3kII\nIYQQLS3RdZxuAZKAtUDhUf+mN3KOEEIIIUS7lOg6TrKlixBCCCE6DQlshBBCCCHiJIGTEEIIIUSc\nJHASQgghhIiTBE5CCCGEEHGSwEkIIYQQIk4SOAkhhBBCxEkCJyGEEKIT2717N5MmTcLpdKLValm5\nciUvvfQSGo2G/fub3tAjOzubG2+8sRV62jFI4CSEEEK0gj179jB79mxycnIwm804HA7OOussnnrq\nKfz+xG2mce211/L999/z4IMPsnTpUkaNGgWAoihxnR9vuxNFq2zyK4QQQhyP3QfKqA5E2rQPZqOW\nvt1Tm3WN9957j+nTp2Mymbj22msZPHgwwWCQdevWcc8997Bt2zYWLVrUQj3+kd/v58svv+S+++5j\nzpw5sePXXnstM2bMwGAwtPg9OzsJnIQQQrRb1YEIToetTfvgcvuadf7evXuZMWMGvXv3Zs2aNWRk\nZMSeu/XWW/nDH/7Ae++919xu1qukpAQAh8NR67iiKBI0HSeZqhNCCCES6JFHHqGyspIlS5bUCpqO\n6NOnD7fddhsAkUiEP/zhD/Tt2xeTyUTv3r2ZP38+wWCw1jnZ2dlcfPHFrF+/ntGjR2M2m8nJyWHp\n0qWxNg888ADZ2dkoisJdd92FRqOhT58+ALz44ov15jgtXLiQHj16YLVaOffcc9m2bVu9r8ntdjN3\n7lx69uyJyWSiX79+PProo6iqGmuzb98+NBoNTzzxBM8//3zsNZ122ml8/fXXda75ww8/MH36dDIy\nMrBYLJx88sn89re/rdWmsLCQG2+8kczMTEwmE4MHD+aFF15o7O1vcTLiJIQQQiTQu+++S58+fRg9\nenSTbWfNmsXLL7/M9OnTueuuu9iwYQMPPfQQO3bs4I033oi1UxSFXbt2ceWVVzJr1iyuv/56/vGP\nf3DDDTcwatQoBgwYwLRp00hOTmbu3LnMnDmTiy66CJvNFjv/p7lL9913H3/84x+ZMmUKF154IZs2\nbWLSpEmEQqFa7aqrqxk3bhxFRUXccsst9OjRg88//5zf/OY3FBcX88QTT9Rq/8orr+Dz+bjllltQ\nFIVHHnmEadOmsWfPHrRaLQBbtmxh7NixGI1GZs+eTa9evcjNzeXdd99l4cKFQM3o2ejRo9Fqtdx+\n++2kpaXxn//8h1mzZuH1ern99tuP/X/OcZDASQghhEgQr9dLQUEBl156aZNtt2zZwssvv8zNN98c\ny3e65ZZbSE9P5/HHH+eTTz5h/PjxsfY7d+7ks88+Y8yYMQBceeWV9OjRgxdeeIFHH32UwYMHY7fb\nmTt3LiNGjGDmzJkN3ru0tJTHHnuMqVOn8vbbb8eO//a3v+XBBx+s1fbxxx8nLy+Pb775JjaCddNN\nN9G1a1f+9Kc/ceedd5KVlRVrn5+fz+7du0lKSgLgpJNO4tJLL2XVqlVcdNFFANx2220oisLmzZtr\nnfvQQw/F/vvee+9FVVW++eYbnE4nADfffDMzZ87k97//PbNnz8ZoNDb5PjeXTNUJIYQQCeLxeACw\n2+1Ntn3//fdRFIU77rij1vE777wTVVXr5EENHDgwFjQBpKWl0b9/f/bs2XPM/fzwww8JhUKxKcMj\n5s6dW6ft66+/ztixY3E4HJSVlcX+nXvuuYTDYT799NNa7a+++upY0AQwduxYVFWN9bO0tJTPPvuM\nWbNm1QqafmrFihVMnTqVSCRS676TJk3C7XazadOmY37dx0NGnIQQQogEORIweL3eJtseyQnq27dv\nreNdunTB6XSyb9++Wsd79uxZ5xrJyclUVFQccz+PXPun905LSyM5ObnWsV27drF161bS09PrXEdR\nlFhC+hE9evSo9fjIaNGRfh4JoAYNGtRg/w4dOoTL5eJvf/sbixcvjuu+iSKBkxBCCJEgdrudbt26\n8d1338V9Trx1k47kB/3U0QnaiRCNRjnvvPOYN29evfc66aSTaj1uiX5Go1EArrnmGq677rp62wwd\nOjTu6zWHBE5CCCFEAk2ZMoXnn3+eDRs2NJog3qtXL6LRKLt27aJ///6x4yUlJbhcLnr16pWwPh65\n9q5du8jOzo4dLy0trTOClZOTg8/nY8KECS1y7yN5Uo0Fl+np6djtdiKRCOecc06L3Pd4SY6TEEJ0\nAJFIFI/Pz8EyH3mFFezYV8rOfaXsyi9jX5GLUlclVf5gwkcbxLG75557sFgs/OIXv6h3Oik3N5en\nnnqKiy66CFVV+fOf/1zr+ccffxxFUZg8eXLC+jhx4kR0Oh1PP/10reNPPvlknbbTp0/niy++YPXq\n1XWec7vdRCLHVrA0LS2NcePG8Y9//IP8/Px622g0GqZNm8Ybb7zB999/X+f50tLSY7pnc8iIkxBC\ntFORSJQydxWl7mqC4Sg6nRa9Toder8NiMaCqoKISCkcpcQeJlFcTjUawmfSkOcwk2UyyXUY70KdP\nH5YvX87VV1/NgAEDalUOX79+Pa+//jo33ngjt99+O9dddx1/+9vfqKioYPz48WzYsIGXX36Zyy+/\nvNaKupaWlpbGXXfdxcMPP8yUKVO46KKL2Lx5M//973/r5DLdfffdrFy5kilTpnD99dczcuRIKisr\n2bJlCytWrGDv3r2kpKQc0/2feuopxo4dy4gRI7j55pvp3bs3eXl5vP/++2zevBmAhx9+mLVr1zJ6\n9GhuuukmBg4cSHl5ORs3bmTNmjWtFjxJ4CSEEO1MMBShqNSLyxfAYDRgs1nqDYBqDiloDRqMhh//\nnIfDEQ6UVhE56CEzxUqa04pGIwFUW5o6dSpbtmzhscceY+XKlSxatAiDwcDgwYP505/+xM033wzA\nkiVLyMnJ4cUXX+Stt94iMzOT+fPnc//999e6Xn11mI5+Lt62R/vjH/+I2Wxm0aJFrF27ltNPP53V\nq1czefLkWuebzWY+/fRTHnzwQV577TWWLl1KUlISJ510EgsWLKhVpbyhe//0+NChQ2NbwyxatAi/\n30+vXr246qqrYm0yMjL46quvWLBgAW+++SbPPfccqampDBo0iEcffbTJ19dSlPY0rKsoyghg48aN\nGxkxYkRbd0cIIVpVNKpSXOalxFWN1WLCZNQ363qqquKrChAKhuiRYSfFYWmhnracTZs2MXLkSBr6\nu99Z9qoTbevIz9l7H65jyLDhdZ7f+u1mJk88C2CkqqqN1jWQESchhGgHKqsD5Ba40Bv0pCY3XfMn\nHoqiYLeaUC1GCsurOFhRSZ9uybVGp9o7CVhEe9NxfnuEEKKTKi7zUlxehdNhRatp+TU7iqLgsFsI\nhyNs31dGtzQbGcnWFr+PECcCCZyEEKKNRCJR9hRUEIzSYqNMjdHptKQm2ylxVeGp9NO7azJarSyu\nFuJYyG+MEEK0gVA4wo59ZahaLUk2c6ve22G3oGp0bNt7iGCobfOHhOhoJHASQohWFgiG2b63FKPJ\niNloaJM+mAx6rFYL2/aWUlkdaJM+CNERSeAkhBCtKBAMs31fGXabpc2TtPU6LU6HlZ35LjyV/jbt\nixAdhQROQgjRSgKhMD/sK8Nht6DT1b9/V2vTajSkOG3kFrpx+yR4EqIpEjgJIUQrCIYi/LCvDFs7\nCpqO0GgUUp028orcVHiq27o7QrRr7XJVXVV1sK27IIQQLSYSibIrvxyr1Yy+nQVNRyiKQorTxr5i\nDxqNgsNmatX7b9++vVXvJ04sLfnz1S4Dp/0lXjyVfpKsrfuLK4QQLU1VVXIPlKM36jHom/8nNxwO\n4XK5SUtLix0rLS3F6XSg0zWv0riiKCQ7bewpdNOvu4LNYmxud5uUlpaGxWLhmmuuSfi9xInNbLGQ\nkprWdMMmtMvAyWG3kFvopk9XWv1bjxBCtKT8YjcRNNiOc/VcJBJlZ345m3eVsLfIxbfb9hBUDegN\nBnRaLVoN+MqLsBsjTJt8DqcO7Ea3tOOvCaXR1Iw87TpQwck9UzGbmheMNaVnz55s3769VXe3FzXK\nXFV4qkOYTW2zsrO1paSmkdW9R7Ov0y4DJ0WhZr690E12VxWnvXVrnAghREsoc1Xhqg6RnHTsVbr3\nFbt59/NcvvyukEp/6McnNFYUIBxRCUfCACiWdHzAS//dxkv/3UaPDDtnDM7i/NG9SbYf+5dPjUbB\n6bCxK7+ck7PTMOgTO73Ys2dPevbsmdB7iNqiUZWte0o42Zn4wqudTbsMnODwfHuyjb1FHnorrT/f\nLoQQzeEPhMg/5CXFaTum87bvLeVfH+1gS+6hep/XaRRC1S5CAT9oNGi0BrSmpFo7zeeXeMlfs4MV\nn+zk7OE9uHTcSXRLO7Z+6LQarDZzTfDUK1UqjHcyJRU+DIYTY6SppbXbwAl+DJ7yCt307ibTdkKI\njuHI9JojyVoroGlMmbuapau+59Nv8msdNxt1DD+pCyNO6sLgPumkOcxs3bqFWbNmxdo8t/h5UjKz\n+W5PKRu2FfLD/nJUFcKRKB9+vY+1m/cz5cy+XDmhP2Zj/FNvBr2OSDRK7oFy+vVMjfu1iPYtEoly\nsLyKlFbY5qczUlRVbes+xCiKMgLY+N6H6xgybHjsuKqqlLt85HRzYJeEcSFEO7f7QBmqRofJ0HSQ\noqoqH2/az9/f2YI/GI4d75pq5aIzcjhnZM9awU5xcTGzZ8+moKAgdiwrK4vFixeTmZkJQKm7mvc/\nz2X1V3lUBX68ZkqSiZsuHsbogd2O6fV4fdUkWXR0z3Ac03mifSoq9eCqirRK8n9HsfXbzUyeeBbA\nSFVVNzXWtkOMvSqKQrKjpkCbbA0ghGjPylxV+ENqXEFTdSDEX17byDNvbIoFTTazntmXnMJTd5zH\n5DE5tYKm0tLSWNCUlZXFkiVLyMrKoqCggNmzZ8cSrNMcZq69cDB/m3cBV0zoj15X86e+3OPnkWUb\nWPzWNwSOYY86u81MuScgNZ46gUgkSklFtQRNzdAhAic4kqxoZVd+Bf5AqOkThBCilQVCYfIPeePa\ntLeozMfdf11ba2ru3JG9+Oud53H+6N5oNXWnxZxOB3375sRGmIYNG8bixYvJysqib98cnM7aI0IW\nk56Z5w3kL3MnMurkzNjxVV/lcc9fP6bgkDfu1+Z0WNlb7JG/vx3cwXIfxhNkFV2idIipuqOFI1G8\nnspWWekhhBDxUlWVHXtLMZpNTRa53JlfzoMvfYGnqqbYr9mo49bLhnPW0O5N3ud46zipqsqHX+9j\nybtbCB4ebbKa9Nzzs9EMyUmP5yXW/P31VjKod7oki3dAkUiUrXsOkSq5TXV0uqm6ox1Z6fHD/lIi\nkWhbd0cIIQAoLvOiKpomg6avdxRx/9/XxYKmHhl2Hv+/CXEFTQA6nb5W0AQ1RSSbKn6pKArnnZrN\nn/7f2fTskgRApT/EghfW8+HXe+O7t1aD2WIir6girvaifTlY7sMko03N1uECJ6hZ6WEymdiZX0Y0\n2n5GzIQQJ6ZAMMzBimrsTUzRfb2jiEdf2RAb8RncO40HZ48jM/XYSgU0R/eMJB66ZRwj+3cBIBJV\neXbFZlaEWkFJAAAgAElEQVR8sjOu800GPYEQlJT7EtlN0cIikSglrmqsZsltaq4OGTgBmIx6NDod\neYXyzUcI0bb2FFaQZLc02mbzzoM8+spXhCM1X/bOHJLFfTeMwWpu/REAs1HPr39+BpPH5MSOLVv1\nPf/8cDvxpG8k2c0UllbKYp0OpLjMi7kNftY6ow4bOAFYTEb8YZWCEndbd0UIcYIqqagkSuNTdN/t\nOcQjy74kfDi9YOyw7sy96tQ23fBXq1GYNWUoP5s0MHbs32t28MrqbXEFT06nldwCl6RMdADhSJRS\ntx+LSUabWkKHDpwAkmxmyjwBylxVbd0VIcQJJhiKUFjqa3QVXf5BDw8v20AwXBNgnDG4G7dfMbLe\nVXNtYdrZ/blh8pDY4xWf7OSNtU1P22k1GkxmI3sl36ndqxltkqCppXT4wAlqlsnmH/Liq5JhYyFE\n69l/0IXN2nDQ5PL6+ePLX1B1eK+5kf27cMdVp7a7FWlTz+zLzRcPiz1e/sE2/vPFnibPMxsNVAVV\n+eLajh0ZbTpRNvJtDe3rt/c4KUpNjafdBa5Y0qUQQiSSp9JPlT+C0VD/zlWBUISHln1JSUVNUNG7\nm4NfXX0aunYWNB1xwel9uPaCQbHHz7/zbZ3tX+rjsJvJP+QlEAo32Va0vqJDMtrU0hL6G6woylhF\nUVYqilKgKEpUUZSLE3UvrUaD/fCGlLLSTgiRSKqqsrfIjSPJ2mCb51d+w678mmmslCQT9/78DMzG\ndr09KJeOO4nLx58Ue/zMG5v4Pq+00XMURSHJbiE3vyKu3CjRekLhCGVePxYZbWpRif7qYwW+AeYA\nCf+NMuh16A168grLE30rIcQJrKjUi96gR9NAntKHX+9lzcb9ABj1WuZfewapjqariccrHI4QCIZj\n/8ItmKD9s0kDOe/U7Jr7RKI8suxLCksbLz2g12lRtFoKD3larB+i+QoOebFaZH/XlpbQwElV1f+q\nqnq/qqpvA62SCWk2GagOqRwskxojQoiWFwxFOOSqxtbAB1JugYvnV34be3zrZcPp3c153PdTVZWq\n6iAV7koq3F683kqikRAmrYpRq2LQqESCATweH+UVXlyeKgLB4582UxSFmy8exin9MgDwVYf440uf\n420ih9RmNXHI7afKHzzue4uWEwxFcPkCmIxN75kojk37Hjc+Tg67haJyLxaTDrtVom0hRMvJP+jG\n2kBCeKU/xJ9e/YrQ4RV0F57eh3Gn9Diu+4TDEbxVfjSopCaZSM5wNvkhqKoqldVBSl1VVLiq0Wi1\n2K3mBkfGGqLVarhrxmncu/hT9h/0UFRWyeOv/o/7rh/TaGK702Fl94EKBvfJOOZ7ipaVf9Dd6MIF\ncfzaZ5ZiC0h22MgtcEuyuBCixVT5g/j84QYTwpe88y0HyysB6NcjmesvGnzM94hEo5S7KwkGAvTJ\nTGJwnwy6piXFNXKgKAo2i5HsbskMycmgW6oFr68St7fqmPOPLCY98687A4e1JrF4S+4hlq3+vtFz\ntBoNRpOBAweltl5b8gdCeP2hBn9ORfN02sBJo1Gw283kHiiXhEUhRIvYW+husEL4+i0HWLu5ZhWa\nxajjzqtPO+YClx5fNV5vJX0y7fTvlYbNcvyroRRFISXJwuA+GWSlWXG5fMdc6TvdaeHumafFak69\n/dluPvu28ZV2FpOR8sognkr/cfddNM/+gx7sbTDaFApHKS6vJK/IzQ/7y9iWV0peoYviMh/VgVCr\n9ydR2mU4+sB980hKctQ6dsnlV3LJ5dOP6ToGvY5gKEJ+sZueXY8/x0AIIco9VUQVpd5yAqXuaha/\n/U3s8U0XDyMjufEtWI4WjkRxeyrpmmIlIyUFRWnZaa5kuxmH1URhqYfSCi9OhxWtJr7vzQN7p3Hj\n5KE8/05N3tZfV2ymV6YjtlFwvfdLsrK30M2gPoZ2V7Oqs6vyB6kORUmxJPbjPRiKsCu/gm17S9mx\nv5yCQ15KXVU0tqjdaTfSLdVGdlcHA3qlcnKv1BZdNBGvt1f8m7dXvFbrmMcT/yip0lqjMYqiRIFL\nVVVd2UibEcDG9z5cx5Bhw1vs3hUuHz262Em2y3yvEOLYqarK1twSnA5bnaBGVVUeeGE9W3YfAmr2\noPvV1afGHfxUB4L4qwP0656C2ZT4RF5fVYA9hS7MFhMmQ3z3U9WajYA/2rgPgB4Zdh6dc3ajU0HV\ngSA6ovTJSmmRfov4bN97CJPZnJB6YaFwlG92HWTdlgP8b3sR/mDzU2F6ZSYxemA3Rg/qRnZmUot/\naYjX1m83M3niWQAjVVXd1FjbhIakiqJYgb78uKKuj6Iow4ByVVWbrqzWQpwOK/uKPVhMeoz6djnI\nJoRoxw5VVKLT6+v9o75m475Y0JSSZGL2pafE/cff66tGr4VBvdNbbWTGZjEyqHc6eUUVeHzhRreL\nOUJRFH5x8TB2F1Swr9hDfomXf7y3lVsva/gLrtlooMLlw+WtxilfWluF2+cnFAVbC/8seasCrNqw\nl/e/zMXlrX+612LS0zXVSpcUK1aTHqNBi1ajUB0IU+UPU+aupqjMh8tX+/x9xR72FXv495odZHd1\ncO7IXow7pQd2S/utPZXoKGIU8DE1NZxU4PHDx18CbkzwvWOOFGjbnV/BgOw0We0hhIhbJBKlqKyS\nlGR7nefKPdW88P53scdzLhuOLc4d6N3eKpJMOnpkOlr9W7ZWq6Fv91QKStyUuXw4HdYm+2DUa7nz\n6tO4+68fEwhF+OB/exmSk85ZQ7s3eI4jqeZLq81ibLcV0zsLVVXZf9CNw25rsWv6qoOsWLuT97/c\nU2ehlc2s59QBXRncJ42B2WlkJFvi+jn2VgXZmV/Ojn1lbNl9iF0HftzrcG+RmyXvbmHpqu85d2Qv\npp7Vl8yUhovMtpWEBk6qqn5CO0lA1+u0BPU68g+66SX5TkKIOBWVeTHVEwypqsrfVn4b24du/PAe\njOifGdc1y10+MpMtdEltuQ+545GV4cBkqCK/xEuy09bkl8ruGXZuungYz7xRM5Px3Jub6ZvlJLOB\n16HRKJgtJvYVucjpLlN2iVTmqkKj1bXIwEAoHOU/X+by+sc/4Kv+Malbo8BpA7txzsheDOubgV53\n7B/vdouBkf0zGdk/k59NgjJ3NV9tK2Lt5v2xICoYivCfL/ewasMezhzSnasmnky3tLpfXNrKCTVv\nZTUbqXD5qPBWS76TEKJJoXCEMre/3tGmL78v5KttRQAkWQ3cOHlIXNescPnommIhI6Vtg6YjUp0W\ndHoNeYVukh1NB08TRvRka+4hPvkmn+pAmMf/+T8enD2+wQ9Rs1FPhTsof3cTKBKJUlDqI9nZ/J+p\nXQcqeHbFJvYV/1gFXq/TMOnU3kw+M6fFR4BSHWYuPKMPF57Rh33FHlZ/lceajfsIhCJEVfhsywHW\nf1fAhOE9uWriANLaIJn8p06owAkO5zsVubGaDBj0x7ZUWAhxYik45MVST4Xw6kCIJe9uiT2+aeow\n7HGUDqjwVNIluf0ETUc4rCZyusHuAjfJzsZX3CmKws2XDGNnfjlFZZXkFrhYtuo7bpg8tMFznEkW\n9hV7sMuUXUIUl3kxmgzNmvINhSO8snob767fHVsZpyhw9vCeXD1xAOnO+FeJHq9emUncdPEwrp44\ngFUb8nh3/W48VUGiUZWPNu5j3ZYDXHlOf6ae2feYS320pBPuJ1hRFOx2C7vzpb6TEKJhgVC4wS0r\n/vXRDso9NXWKRvbvwpghWU1ez+2tIs1ubPPpuYbYrSb6dXficlc2uVG62ajnzhmnxYKgd9bn8vWO\nogbbK4qC1WJib1FFg23E8QmGIpS6/VjNx1/zq7DUy68XfcLKdT8GTb27Onh0ztncdsXIVgmajma3\nGLhiQn+eu3sSM88biOXwatNAKMKyVduY+5eP2LTzYKv26WgnXOAENfWdFJ2WAtmQUgjRgIIST71b\nVuwr9vDu57kAGHQaZk0d1uQ3fV9VAKtRS7f0hmsftQc2i5Gcbg4q3L4mv1j26ebkugt/rIz+1zc2\n4/Y1XGDTZNRTFYxS4a1usf4K2H/Q1eAWQPH47Nt87npmLXmFNXWM9DoN15w/kEfmnE1OVnJLdfO4\nmI16rpjQn0V3T+LC0/twZBa5qKyShS9+zsPLvoxV6m9NJ2TgBGCzmCh1B/BKdVshxE8EgmG81XW3\nrFBVledXfhMbkbl8fP8mcz78gRAaNUJ2B1mUYreayM5MoszV9EbpF53Rh1En1yTEuysDLHprc6MB\nl9NuYX+xh3Ak2mL9PZF5K/1U+SPHtbVKNKqybNX3PPmvr/Ef3hQ6K83Gw7eezeXj+7erKVWb2cBN\nFw/jsf87hwG9UmPHv9pWxC//8hHvfZ7b5ChpS2o/70wbSHZY2VPoll9iIUQtDW2Qum7LAbbtLQMg\nM8XKpeP6NXqdcDiC3++nb/eWrwaeSE67me5pNiqaCJ4URWHOZcNJOlxzZ8Ph1VHhcIjS0tJabUtL\nS4lEwlitZvIKZcquuVRVZW+xm6SkY59Gqw6EeHjZl6z4ZGfs2IQRPXns/ybQu6ujkTPbVu+uDhbe\nPJZfXjkSp71majIYirDk3S38fsm6Vht9OqEDJ41GwWIxyS+xECLGHwjh80cw/KRYbiAYZul/f9zk\ndtbUoY0uMFFVFbe3ipN6pHbIbUfSk62kJhnx+BqfWnPaTdxyVCHMv7+zhV/Nu59Zs2ZRXFwMQHFx\nMbNmzWLevHloNSr+kEq5uyqh/e/sDlVUotXq4t465wiX1899z6/j6x01/280GoVZU4byf9NGYOoA\nmwIrisL44T155o7zuGB079jx7/JKueOpj1i1IS/h+csd77e5hZmMegJhlZKK1p8nFUK0PwdKPNht\ndVfSvb1uN6XumiBiRP8ujGyiZpPLXUmvzKQOvUN9VoYDkw6q/I1vDnz6oG6cPbwHANWBMPmhHhQU\nFDB79my+/fZbZs+eTUFBAbt35+JyuXHYzewv8RIKN3/LjhNRKByhsKwSexxV349WXF7JvYs/ZU+h\nCwCrSc9vrxvD5DE5HWpEFGoqld98ySn8/sYzSXPWvA/+YITFb3/DAy+sp9SVuMD8hA+cABx2C4Wl\nPgKH53mFECemhkabytzVvHl4WkOjUbj+wsZrNvmq/Dithk5Rt6h3txSioXCTfx9nTR0Wq7Gj2LrS\nfch5FBQUMGvWLAoKCsjKymLx4sWkpaWhKAo2q5m9hz/AxbHZW+iqdyq5MfuK3dy76BOKD09npTrM\nPDh7HKf0y0hEF1vN0L4Z/Pn2cznv1OzYsS27D/Grp9fw1bbChNxTAqfDHHYLuw9USIkCIU5gDY02\nvbL6ewKHt5y4YHRvumc0XMU4FI6ghiP0yGy/uSLHQqNR6NsjhcrKaiLRhvNBrSY9t10xMvZYnzUa\nnTUt9njBggVkZv44Smc06PCHVUpltP+YuLzVVIejxzSSmVfo4v6/r4vtE9cjw85Ds8fRo0v7XuUZ\nL4tJz62XDee+68eQejh491WHeHjZBp5f+W2d7WKaSwKnw3Q6LRqdjgMlUqJAiBPRkdEmjaLWSmzO\nLXCxdnPNnuQ2s56rzj250eu4PVXkdE/ucFMfjdHrtORkOXG5Gg9yhuSkM+XMHAAiUZW0IReDUvMx\nc//998dyno5w2M0cKPW1+AdbZxWJRNlf7MFhiz8hPLeggt8tWYe3KghAv+7JLLx5HGmtXJupNQw/\nqQtP3n4Opw/qFjv2ny/3MO+5tS362S6B01FsFiNlHilRIMSJ6ECJB7NJx7x582olNi9ZuSnWZtr4\nfo1WCHd7q8hKt3XovKaG2CxGuqXbcHsbzx25YEQX1EDNFJwhqSs/n/soWVlZsZyno4NSRVGw28zk\nFpQntO+dRX6JB5PZGPd+dHsKXfx+yfrYfnP9e6bwuxvPxG6JbyPqjshmNnD3zNOYfckpGA5vA7Sv\n2MNdf13LB//b2yKzShI4/USyw0peoZuIlCgQ4oQRCIapDESoqvSxe3du7EN+xaov2JFfUxhQDXo5\n7aSGCwIGgmGMWoWM5Pa3m3tLyUi2YtZrqPYHG26TnkKWkoeq1vwNXbfdxfw/PE5WVhZ9++bgdNae\nwjTodUTRcLCs6bpRJzJPpR93VQizKb6gJ/+ghwX/WE/l4U2oB2ancv8NY2JVuDszRVE4f3RvHpkz\ngR6Hp9WDoQjPvbmZx//5v9h7crwkcPoJjUbBJFsDCHFCOVDiwWoxkZaWxuLFi2MjJP9455tYmxun\nDqdrZv2JtKqq4qusJrtbxyhy2Ry9uyUTCAQIN7AiTqfT8+eH72PK6b2Amim75R/vY9Hi53nkkUfQ\n6ep+cCfZzBSVV+IPNO8DrbOKRKLsLXTjtMc3vVZU5uP3/1iP5/D03Mm9Uph/3RjM9Wwf1Jn1ykzi\n0TlnM+m07Nixz7cWcOfTa/hh//GPckrgVA+zUU9VUOqMCHEiCIRqVwnPzMxkwYIFWDIHYkiqSWbO\ndBqYPHZwg9dwe6vpkW5v041HW4tGo9Cvewpub1WD0x46nZ5rJ48g+3Axxf0HPXz07aF6g6YjHHYL\nuQUuWaBTj71FLkwWU1xTdOUePw/8Yz0V3pqUk5ws5+GgqfNNH8fDaNBxy6XDuWvGaVgPj7aVVFTx\n2799ysp1u47r500CpwY47Gb2H/RI0qIQndxP96QrLi7m/vt/h6Pv2T+2+eYdSkrq31Q0EAxj0EJq\nJ0y2bYjJqKdHuh13I/vO6XUabr9iJDptzYf9W5/uZGd+w9/yYwt0DsoCnaOVu6uoDEbiGi2q8of4\n40ufU1JR86W/Z5ck7r9hTCxgOJGNGZLF47edQ/+eKUDNSOiL73/HQ0u/jCXOx0sCpwYoioLdbiG3\noFy+AQnRSQVDETxVP442lZaWMnv2bFykorfU5DOpvkIKdn5dJ7H5CF9lNb27te1mqG0h1WnBYtDg\nDzY8vZbd1cH0cwYAEFXh6dc2xso61MdmMVLmlQU6RwRCYfaXeOOaoguFozy6fAN5RTU5eelOC/ff\nMKbRxQwnmoxkC3+4aSyXjz8pduzrHcXc+cwa9hXHH7BL4NSIWNJiuSQtCtEZFZd6sVp+rNvkdDrI\n6duXlH5nx47ddcOkBhObPb5quqVaG916pTPL7uqkusrfaH2ny8b1o2/3msCyoNTH8tXfN9gWZA/R\nI1RVZXd+BUl2S5OlLaJRlb+u2MSW3YeAmrIZ910/hpSkjl+AtaXptBquOX8Qv73ujNjqwlJXNc+u\n2NTEmT+SwKkJSTYzxWVVVDczC18I0b6EI1EqfAFMR02B6HR6zrviVhSDDaipC3Pm8H4sWbKkTmJz\nOBKFaJT0TryKrilarYY+3Zy4PA3Xd9JqNdx+xQj0h5eGv/t5Ltvy6o7cHXFkD9E9B07sEgX5xW60\nel1ceXPLVn/Pp9/U1Boz6DTce+0ZjRZpFTCifyaP33YOA3qlAhA5hpklCZzi4HBY2V1QTjQqU3ZC\ndBYHy7yYTLWnMULhCCs+3R17fPXhYpdpaWl1Eps93ip6d3N0qkKXx8NmMZLhMONrZHqte0YSM84b\nCICqwtNvbKI60PAWLiajnlBUOWFLFJS5qnBVhbCam55me+/zXN76dBcAGgXuuPpUTj4cDIjGpTnM\nLPjFWbWm7uIhgVMcdFoNBqOBAwfdbd0VIUQLiESiHHL7sZhr18T58H/7KDu8ke+okzPp1yOl3vOr\nAyHsZh2WOGvqdHZd0+yokUijm/ZOPbMvJ/eqeT8Plley9L/fNXrNJLuZorJKqhqpGdUZVfmD7D/k\nxZnUdF7Thm2F/OO9LbHHN118CqMHdmvkDPFT2sNTd7+YOizucyRwipPFZKSiKoTbJ0mLQnR0h1yV\nGI21g55AKMIbn/wQe3zVuQMaPL+qyk/PzM5fsyleiqKQ0z0Zt6fhEi5ajcL/TRsZywf774Y8tuwu\nafS6TqeV3QcqTph8p1A4wu78CpId1iZHMnMLXPz5X19zZIbpign9OX9071boZed0cs/6vyTVRwKn\nY+C0W8grkqRFITqyaFTlYHkVtp+sNvrgqzzKPTVfjE4b2JWcrPoDI+/hhHCdVv58Hs1o0JGVbsPT\nSImCbmk2rr1gUOzxMys2UdVI/qhWo8FsNpF7AuQ7RaMqu/aXY7Ga0Woa/9kqc1fz0NIvYisUx53S\ngxkTGw70RcuS3/xjoNEo2Kxm8gqkqrgQHVWpuwqdvnYxwEAwzIpPdsYeNzTaFI2qRCLhEzohvDEZ\nyVZ0GpVAsOH8pQtG92FwnzSgZjXTC+9vbfSaJqOeCBoKD3Xe+k6qqpJ7oBydQd/kPof+YJgHl34R\nC/L790xhzmXDT/hcu9YkgdMxMhp0BCIqJRWN7xIuhGh/VFWluMyH3Wqqdfy/G/Jw+QIAnDG4G727\nOuo7Hbevip5dJCG8MX2ykvFVVjdY/06jUfi/aSMwHQ4QPvp6Hxt/KG70mnariUNuP65GRrM6sv3F\nbsKq0uQ+dNGoyp///TV5hTX5tl2SLfz6mtNP2HIYbUUCp+PgsFsoLPXJvkpCdDAunx+tVlcr8KkO\nhHnz05rRJkUhVrDxp0LhCEatgsNmqvd5UUOv09Izo/Gq4hnJVm6Y/OMWNs+9uRlfdeNJ4MkOK3uL\nPZ3u727hIQ9efxjbUcF8OByqU2y1tLSUl/+7la+2FQFgMeq499ozcNikwGVrk8DpODnsFnYXVEiJ\nAiE6kMJDvlofUACrv8rDU1nzoX3mkO70ykyq91yvr4pemfWPRInaUhwWzAal0SBn4qhshp/UBajZ\nX+3v72xpsC3UJKA7kqzszC/vNHmmxWVeSr0BHEdVBg+HQ8ybN49Zs2ZRXFwzEldcXMwv7nyIlety\ngZpRu7tmnkaPLvX/rIrEksDpOOl0WnR6vZQoEKKD8FT6iUCtjVKDoQhvr9sVe3zlhP71nusPhLCb\n9bWKZYrGZXdNprLK3+CUnaIo3HrZcCyH91H79Jt81n17oNFr6rQaLBYzu/LLOvyX1pJyHyUV1SQn\n1c6Xc7nc7N6dS0FBAbNnz+bbb79l9h2/Q+l6RqzNL6YM5ZR+XVq7y+IwCZyawWqWEgVCdBSFpT7s\n1tpbUHy8aT8ub01u0+mDujX4Db6qyk+PLjLadCx0Wg29MpNweRsuUZDmMHPT1KGxx4ve2txk/qjR\noEOj03XofURLyn0UVVST7LTVeS4tLY3FixeTlZVVEzzddjd0H4+iqcljmjwmhwtO79PaXRZHkcCp\nmZx2C3uL3I0WfhNCtK1qf4hgWK1VQiAcicZymwCmnV1/9eAqf4BUhymurS9Ebcl2M1aDlupGpuzG\nD+/JuFN6AFAVCPPkv74m0sRUnMVkJBxVyC/ueCP+RaUeiiuqSXE0vDIzMzOTBQsWoNGbSR9xFVp9\nTcA/sn8Xrr9oSGt1VTRAAqdm0mgUrFYzuQc67rcfITq7gkMerD+p27RuywFKKmpGQ07pl0FOVnK9\n5/qrg3RNlX2/jlevrk6qqvyNTq3ddPEwuiTX5Pn8sL+c1z7+ocG2R9isJjz+MAUlHSd4OlDiptQT\nJLmRoAlqcpru/93vSRs2Db2lpjCj6i/nZxN6odXIis62JoFTCzAadETRUFTqbeuuCCF+IhAK4wtE\nMBxVuykaVVmx9sfRpivOrj+3yVvpJzPFilaKXR43nVZDdmYS7kam7KwmPXOvOjWWf/b6xzvYvrfh\njYCPcNgtlPtC7T54OlKnyV0VbnIrldLSUmbPnk21YximlF4154eqKfzqFX5525w6q+1E65O/Bi3E\nbjNz0FWNt5GNLoUQra+41Fdns9QN2wo5cKjmi86AXqkM7J1W5zxVVQkHQ1LssgU47Wasxsan7Pr3\nTOHqw4VHoyo8+e+vmyxRAOBMqgme2muBzHAkyg/7SgmpCkk2c5PtnU4Haf3PxpZVs3eaQa9l3jWn\nkZlqp2/fHJxOybVraxI4taAUh409hZLvJER7EY5EqfAFaq2GU1WVN44abZrWwEo6b6Wfrmm2Wqvw\nxPHL7uqkuokpu8vGn8TA7FSgpqr4ore+iSsFoiZ4CrK/yNVi/W0J1f4Q2/IOodUb6gTvDflqewkV\nuuzY49uvGMnpw/qyZMkSHnnkEXQ6WdnZ1iRwakFH8p32FFRIvpMQ7UBJuQ+zuXY15s27SthTWPMB\n26ebk+H9Muqcp6oqkXCYNGfTO9SL+Gi1GrK7OnB7Gl41p9UozJ0+CuvhEgWfby3gw6/3xnV9h92C\nLxAh90D7KFVQ5qrih/xy7HZr3GUsduWX89RrX8ce/2zSQMYMyQJqVttJ0NQ+SODUwowGHRFVabfD\nxkKcKKJRlUPuaiym2t/0V6z9MfF42tkn1bt9irfST1aaXbZWaWEOmwmbWUd1oOEpuDSnhTmXD489\n/vs7W8gtiG8kyW4zE0bDD/tKCYbaZuQ/EomSe6CcgvJKUpy2uDeDPuSq4qGlXxIM16wonDCiJ5eP\nr3+lp2hbEjglgN1mptQTkPpOQrShUncVen3tb+jb8krZtrcMgKx0G6MHdqtzXjSqEo2ESXE0nY8i\njl2vTCf+qkCjo0JnDM7iwsO1ikLhKI8t34C3qul8J6gpVaA3GtmWV4qnlXNOvZV+vt9ziIiiITnJ\nGnfg7asO8seXvojtlzgwO5VbLpWNe9srCZwSJNlhJa/QRSDU8C7hQojEOLKZr+0nJQhWfPJjbtPl\n4/vXm7/kq/TTPSNJPrQSRKvVkN3N0WhhTIDrLxpMvx41JSJKKqp46rWv456CMxp0JDtt7Cn0kF/s\nTvjUXSgcIfdAOXuKvTgcNszGxjfrPVogGObBl79g/8GaWYquqVbmXTMavU4+ntsr+T+TIIqikJRk\nZVd+ebuYbxfiROL2+dFotbWCn71FbjbtPAhAutPC2GHd65wXjapEo2GcspFvQiVZTTgseqr9DY8i\n6XVa7ppxGnZLTRCy8YeD/POj7XHfQ6NRSE22URWO8t2eElyNbDp8vCKRKEWlHr7PK0XV6EhxWI9p\nMbxQCQEAACAASURBVEE4EuXxf/6PHfvKAUiyGph/3RjsluZt3BuORHH7qqlw+Sh3efF4fLF/breP\n8govLnclvio/kWjn2PevNemabiKOl16nxWAwkFdYTk731LbujhAnjIJDPmw/Wfp99J50F5/Vt97c\nE1+ln6x0yW1qDT0ykvg+7xAGgw6tpv7v8OlOC3dcdSoLX1xPVIXXP/6Bnl2SOGto3aC3IRaTEZPB\nwP5DPopKfXRLt+NoZmAcjkQ5WOblkNuP0WggNfnYC6RGoyrPrtjE1ztqNvI1G3Xcd/2ZdEuruw1L\nPFRVpbI6QDAQwmLS0S3FjN1irLfivaqqBEMRvJUBSj3VBIIRDEZDnRFaUT8JnBLMbDLg8VVTeMhD\nt3TZyVqIRKusDhJRqfVhfMhVFdtA1mbWc+6oXnXOOzLalGyX3KbWoNVq6NPNye4CN6nJDQcLp/TL\n4NoLB/Pi+98B8MzrG+maam2w0nt9NBqF5CQrkWiU/SU+lBIPaQ4zyUlmjPr4PgYjkSjeqgAHyyup\nDkYwm48vYDpi6arvWLs5H6gpEvrra04nJ8t5XNfyVfoJBUN0SbGS3j25yVEvRVEwGnQYDTrSkq1E\nIlFKXVUcLPei0+uwWU3y5aEREji1giSbmRKXD4tJj1P+KAuRUAUlHuzW2iMK767fTeTwlPlFZ+Rg\nMtT90yejTa3PZjGSajfiqwo0Otox9cy+7D/oYc3G/QTDUR5a+iUP3TKe9GMsF6HVaHAmWVBVFVdV\nkBJXOQpgMmixmfWYjXo0igJKTaAUDEWpCoSo9IcIR1R0Oi1WiwmLtXlZLm9+upO3P9sNgEaBX101\niiE56cd8nWAojNdXTdcUKxkpycf9s6vVauiSaiMjxUqpq4rCUh9miwlznGUUTjSS49RKUhxW8oo8\nVPsbrpwrhGieQDBMdSiK7qjpCV91kA/+txcAg04TW611tGhUJRKR0aa2kJWRRDgUJNzIxr6KojD7\nklM4+f+zd+fhcd3V4f/fd/Z91zraLMl7Esdx9oSEbGyBJgTCWgol7deQFvor/YJLS1MwXUhpKZQW\n6v4wO4QlDQQIEAhkd2IncZzY8SpbtmXZ2mff79z7/WMs2Yq2ka3Rel7P4+exxnPnXsvynTPncz7n\nNJfmtg3Fs3z2G9vK3mk33us57Vb8Xhc+rwuz1Uo8q9E9mKFrIMXxviSnIlmiGRVNMeL1uAj63Xjd\njrLbC0zk4e2dfOfXr4x8vfH2i7nygvC0XyeWSKPm86xpCVETdM1IwK8oClV+J2tbq7AYdIaiSanR\nHYcETrNEURT8PicHuobmrL/IXCsWNdLZPPFkloFoit7BJL2DSfojKYZiaVKZ/JL93oiZURrmOzrb\n9OvtnWTzpZ+rGzc043WNzWwk05JtmisGg0J7ODBpY0wo1Yxueu+V1AVLI3BO9Cf45+88Q24G7hlG\ngwGHzYLHZcPjsuN1O3A7bThsllEzDs/Xw9s72fLgrpGv3/O6Ndxy2bJpvUZR0xiMJKj22ljZHMJi\nHlvDdL5MRgMtdT5a6zxEY0lyedkdfjZZqptFRoMBj9vBgeMDrGmpWvSDQ/OFIvFUlkgiSzZfpKjp\nGI1GFIOCQTGMTPnWdL3Uqbmooesauq5jMRnxOi343XbsNkkXi6kV1CLxdIGg/0zglC8U+eW2w0Bp\nSeQPrm0fc9xwl/CAR7JNc8VuM1MXcDKYzI5ZZj2b12Xl7/74Gj751ceJpXLsPzbEv923g4+/Z/5v\n33910HT7dct52zQbXA4vzbWHfbNSyO122ljbauHwiSESBXXSf5ulZH7/pC1CZpMRm83Gwa75MRZg\npqlFjb6hJK8c6WffsQH64zlMFis+bynV7fM48LrsuJ1WHHYLDntpJ4fbacPnceD3ugj43DiddhI5\njUPdUV4+3EdXT0w+9YhJ9Qwmsb9qHtjju46PNBW8cm2Y2uDYIuRkKkv9DC11iHNXE3Rh0ItT/j+v\nDTj51AeuGqlTe35/D/92345Jl/rm2k+fODgmaHrf69dO62cumyuQTmdY0xKa1d1vJqOBlc0hPHYj\nkWhy1s47n0ngNAdsVjNGk5mOrsFFM9Mumytw+MQQrxzpZzBZwO12EPC5cTls51QToCgKDpsFv9eJ\n3+siW4T9x4fYf2z2uwGL+a9Y1BiMZ3HYzjQeLGr6SAEuwG3XLR9znK7rqAWVoMykmxdawwGSqcyU\n98W2sJ9Pvu/KkWWqHftO8YUfPDfvgidd1/n2r/bw7bNqms4laMrk8qj5PKtbqiqyNFeOhmovdUEn\ng5HEovzQPx0VD5wURfkzRVE6FUXJKIryrKIol1X6nAuB3WahqBg50j20oIOndDbP/mMDHOiKlBrA\n+d24HNYZ//Rus5oJ+FxYbTaO9ibY19lPQgIocdpANI31VTuAntt3ipMDpU/IF7SGWN4wdvt6Mp2j\nNlj+aAxRWRazkeZaD9H45F3FAS5sq+Jv3nclltNLdM++cpLPfedZsvMkM11Qi/zn/+7kp0+e6R/2\n7ptXTztoSmdz6KrKyubQeRemn68qv5PWeu+SLxqv6L+CoijvBP4N+HtgPfAS8LCiKKFKnnehcDms\n5IsKR7ojCy54yuYKHDw+SEd3DKvVWgpqxtniPdNMxtIMKJvDTmdPgoPHB2WszRKn6zq9QymcZy3T\n6brOT544M17l9uvGryUpFAqEfM6KX6Mon99tx+Mwk87mpnzuRe3V/PX7rhypb9p5sJd7vvYUseTU\nx1ZSNJnjnq89xaM7jwOgKLDxtou588ZV08s0ZfNQLLK8MTitjuSV5HHaWN7gIxJLLtmu45UOX/8S\n2KLr+rd1Xd8PfAhIAx+s8HkXDJfTRl5jwWSe1KLG0VNRDnQNYbKUltJM43SmrTST0YDf68RoNrPv\n6CAn++ML4vsnZl4kkcFgGj1eZd/RQQ51RQBorvWwfnn1mONSmRzVPvu8eUMSZzTVeMnn8qjq1Dvm\nLl5ewz1/fA2O05tIOk5E+Jstj3OsJ17pyxzX4e4on/jKoxw4XhqjYjEZ+Kt3Xc7rr5je7rlMNo+m\nFuZV0DTM5bCWgqfo0gyeKhY4KYpiBjYAvxt+TC+9sz0CXFWp8y5ELoeNvGag48T8nWun6zp9kRR7\njvSTK0LA5x63lf9ss5hNBP1uYhmVvZ0DUkC+BJ0cSOJ2jt4Rd/byyO2vWT7up/xcNk/1JB2rxdwx\nGBRWNAaJxtNlfSBauyzEP/6f1xDwlHZ9nRpM8ddffYzHd3VV+lJHaJrOg08e4pP//RgD0dJcvIDH\nxj9uvI6rL5xen6ZMrkBRLbCiKTTvgqZhTruVFY1+ItHUvH3fqpRKZpxCgBHofdXjvUBtBc+7ILkc\nVjTFwIHjA/OuwDGdzbO3c4D+WJaAzzWqAHe+cDlsOBw29h0bpC8yeT8YsXjEU1l0DKPeXI73xkfm\nf4W8dq4ZZ65ZOpsn4LEt+pYgC5nVYqKp2k00MXW9E0BzrZd//tD1LKvzApArFPnSj57nKw/sJFXh\nxsN9kRSf/eY2vvWrPajFUhCxvNHPv9x9w7RGw0CpiWshl2fFPMw0vZrTbqUt7CUSSy6pjL/cNeYR\nh82KyWxh39H+eVG3UyxqHD8V5UBXBIfTjsdln9dFtCaTkaDfTV80w+F5nL0TM+fkQBLXq3rLPHhW\ntuktEwzzzWZy47YmEPNL0OfAYzeTypRXs1Tlc/BPH7qemzacmUX4yPPH+IsvPsL2vSdn/PoKapH7\nHz3AR7/4O17q6ANK9UxvvW45//Cn141kwKbzeul0hhVNgQUT1HucNlpqPQwuoVYFlazmHQCKQM2r\nHq8BeiY78DN/twmPxzvqsdvuuJPb7njHjF7gfGSzmjEaDew7OsiyOu95T/E+V5FEhuM9cex2K6Hz\nGGQ5F7xuB5lsnlc6+1jRFCx7iKdYWNLZPDlVx3nWG8xgLMOTL5WWZ1x2Mzdf2jLmuGyugNc1/tR4\nMf8013rZ1zlA3mQsq4u31Wzkz952CauaA2z9xW6yeZWheJZ7v7udC1ureNfNq1jdcn77kwpqkcde\nPM4Djx+id+hMhjvgsfHRt2/govaxNXVTUYsaiUSa1S2hBfez6XPbCasaPZEUfs/832zx4AM/4sEH\nfjzqsXg8VvbxSiXTa4qiPAts13X9L05/rQDHgf/Qdf3z4zz/EuCFhx55igvXra/YdS0EmqYTjacI\neazUV3lmLdOTy6scPRUlXwSve/YzTKpaIBqNEQqdubENDAzg83kxmabXQbygFokn0rPWZVfMrkNd\ngxjNllFvMt/+1Z6R+qa3vXYF733d2jHHDUaTrG4OSEC9gOQLRfYdHcDndU1r+ao/mmbLT3ex8+Do\nipELW6u4+bJmLl9dN63dwD2DSZ586QS/ee4og7HMyOMGg8KbrmrlXTetHilSnw5N04lEE6xqDmJb\nwIN1u3piJLIqbtfC68K/+6UXufXmawE26Lq+c7LnVvrO8QXgm4qivADsoLTLzgF8s8LnXfAMBoWA\nz0U8lSXaOUBb2FfR/1DFosbJ/gSDiSxulx3HHLypqGqBTZs20dFxmC1btlBbW0tPTw8bN26kvb2N\ne++9d1rBk9lkxO91cag7SkuNB7+M1Fg0cnmVVE4laD/zb5rKFnh4RycAZpOBN13VNua4fEHFZTVK\n0LTAWMxGWuu9HD4ZIziNDHiVz8Hfvv8qnt7dzfd/s5ee09mh3Uf62X2kH7vVxLr2alY2BVje4Cfo\nteN2lILxVCZPIp2nqy/Bwa4h9h4dpONEZMw51rVX80dvvGCktmq6dF0nEkvS3uBf0EETQEONh44T\nQ2SyeezzsBZ2plT07qHr+o9O92zaTGmJbhfwel3X+yt53sXE5bShqkX2HxuiymejLuSZ0YJBTdMZ\niKY4NZjCbrdO66Y006LRGB0dh+nu7mbjxo1s3ryZe+65h+7u7pE/PzsTVQ6DQSHoc3GsL0FBLVId\nkLqWxaC7P47LMToQ/s2OTjK5Um3ga9c34XePXeZOprKsGKcRppj/3E4b9aHitJeDFEXh2osauGpt\nPY/t6uL+Rw+MLK9lcirPvnKSZ1+Zfv3TpatqufOGlSxvDEz72LMNRpMsq/Usiqy4oii0hQPsPdqP\nyWRccEuO5ar4xy5d178CfKXS51nMTCYjwYCbeDpH/+E+6gJOqvzO8wqgNE2nP5KidyiFyWzC75u9\nWV26rpPNq2RyKpoO6Dp2m5lgMMiWLVvYuHEj3d3d3HXXXQCEw2G2bNky7aBpmKKUgqeeSAq1qFFf\n5ZnBv42YbeMN8y2oRX7xdGmYr6LAba8ZO8xXLWpYzQYZGr2AVfudZLMFkuksLsf06j+NRgM3bWjm\nhvVN7DncyyPPHWFnxxDpaey4a6nz8pp1DVx7UQNVMzCmZyiapCHkwudePNlwg0FheWOAfUcH8U9z\naXWhkHz1AuJyWHE5rAylsvREUvicVqr9zmm9EaQyeXqHkiQyBSwWM74KB0yRRJZ9RwfpPBWl81SM\nk/1JIoksucLYxnZmkwGfy8bKmz9M8ulHyMdPkR06xubNm6mtPf8OFn6vk6FEmmIxRmPtuaXVxdwb\nf5hvF5FEaQTPFWvqqQ+NzZwmUhmW1SysjQ5irMZaLx1d574cpGkq3/rvf6Gj4zBf/ep/UzA4eH7P\nUb7z44dw+0K0Ll9NoajhsltwOyyEvHaWN5aW8ryumcsKReNpqn02qvzzv5h6uqxm08jS6kLbXFQO\nCZwWIJfDBg4bubzKoe4ooGO3GPG5bFgtRowGA0aDQlHTUVWNbF4lmsySVYsoigGn3UrAV5nderqu\nc7BriG17TvLSoT6O95bfvbegavRH0/RHwdt6zenX0/j015/hjtepvPm6teMuv0yH1+0gnsjQ1SPB\n00JULGoMxbMEzroZa5rOg0+c1fBynGG+mqZjpLTcIxY2RVFoawhw4NgAOYNh2qOezi4J+PCHP8Tm\nzZv59pdLJQHhcJiPfPyOc85ulyueyOBzmKkLLd7st8dpoz6gMhDP4FlEGTWQwGlBs1pMIzcNtagx\nkMhTLJYyOZoOBqV0kzEZjdjsdhwVTJn2DqX47XNHefKlE/RHJ25YZ7eaCHhs+N02nDbzSLYrnS0w\nEE3R3RdFMZ75FKkoBnDW8JOnj/PgtuNcuqqON1zZyrr2qnPOlHncdmKJNEpfjIZqCZ4Wkt6hJNZX\nZRme33+K7tPDfNcuC7FinJqTZCpLfZXUty0WBoPC8qYgezv7MRgc06qlCYVCFSkJKFcilcVlMy6J\nD241QRfxdI5MroB9gRe+n00Cp0XCZDTMenGhruu8eLCXh545wq5Dvby6s4VBgbawnwvbqljRFKCl\n1kuVb+IWB6pa4BObNnH4cA93/+WnOBkrsuOVbnojpW2/mg479p1ix75TLG/wc+eNq9iwsuacAiiv\n20EknkYhRliCpwVB03T6YxkCvtGp/59MkW3SdZ1iUcU3Rz3RRGWYjAZWNYfYf2wAl2t6wVNtbS2b\nN28eCZqAGSsJmEwylcVuVmiu81X0PPNJa72fVzr7MZuM4zajXYgq2sdpuqSP08JQ1HS27T7BT544\nxNFTo5uGGQwK69qruebCMJetrsPtmF4Nwnh9nF451MWLh6M8tusEQ/HsqOevbg7yJ2+5iGX153Yj\nisRT1Hjt1EgX6XmvbyjJYDI/qih437FB/nbLEwA01Xj494/eOCaQTqSyhDylekCx+OQKKgeODuJ2\nO8oeOD7c5mR4xy6cyThVKnhKprJYjNAa9s/rCQyVkMkW2N81NK/rneZTHyexiGiazrOvnOS+R/bS\n3T+6vX6Vz8HrLm/hpg3N+M6jDslkMo9Jla9d3sja5Y28+5a1PPvKSe5/7MDI5PN9xwb5+H89yi2X\nL+MPX78W5zR3TPk9Tk5FkpiMBoIzsEtGVIau6/QMpvD5Rge4P33i4Mjvb5tgmG+hUCDklRYEi5XV\nbGJlS5D9p4OnqTJPAwMDI0FTOBwe1fZk48aNbN26dcaX6xKnM00tdb4lFzQB2G1mwiEXfdE0XvfC\nv89K4CTK8sqRAb7+y5fpPDk6w9Te4OeO65Zz2Zp6jBXedmo0GrjmogauuiDMjr0n+c7Dr3BqMIWm\nw8PbO3lhfw8fefsGLmyrmtbrBrxOuvoTmEyGORtxIyY3GE1jNJtGvemc6Ivz3L7S9Kag18614w7z\nzVHltS/KLdHiDKvZxKqWIAeODeJw2CctGPf5vLS3l5qjDmeYhmue2tvb8Plmduk+nsjgshmX1PLc\neKr9TmLJLNlcYcE3+pTASUyqL5Lm27/aw7Y93aMeX9Uc4N03r+GC1tCsf4IyGBSuvCDMhlW1/Pzp\nw9z/6H6y+SIDsQx/v/Upbr26jT96w9qyax4UpdSl/cjJGCubDDgWccfbhUjXdU4OJvF5R2ebHnyy\nY+T3b7mmDbNpvGG+edrqphdIi4XJajaxuiXEweND6OjYLOO/OZtMZu69995RJQG1tbVs3br1nEY7\nTaTUETxFldcmveNOW1bvZ++Rfszm0u7vhUoCJzGubF7lJ48f5MEnD5FXtZHHl9V7ee/r1rJ+efWc\np5zNJiN3XL+C16xr4D/v38nuI6WG9A9tO8zB40N8/D2XEypz+U1RFPw+Jwe7IqxukcHA80kkkcFo\nGp1tGopneHzXcQAcNjO3XNYy5rjhYb6LpSBVTM1sMrKqOcjhE0Mk8hPPTBuvJGAml+dKs+eSNFa7\npQTgLCajgdawj0Pd0Xld7zQVuaOIUXRdZ9vubj7y74/w40cPjARNXqeVu+9Yz7/cfQOXrDi3nWyV\nUuVz8PcfvIa73nzRSNbh0IkI//c/H+Xljr6yX8doMOBxOzh4fBC1qE19gJgV3f2JMV2if7HtMGqx\ntLHlDVcsG3ercyqdpS4kRf9LjdFoYHlTEK/DxGA0yWxvgMrlVSKxJG1hrwRN43A5rFR7bSTT2amf\nPE/Jx2oxYiCa5n9+9hLP7+8ZecxoULj16jbuvHHVtAuvZ5Ph9HWubgny+e9tpzeSJp7Os/mb2/jQ\n7Rdz86UtZb2O2WTEZrPR0TXIyubZX4YUo0USGRSDcVSNUipb4DfbzwzzvfXqscN8C2oRl80kmcMl\nSlEUwtVenPYMR3viuJyT1z3NlEQygwGNtctCi3ZO20yor/IQ7RygoBYX5PdJMk6Coqbz0LbDfPSL\nvxsVNF2yooYv/sVNfOBNF87roOlsrfU+/uXPSlkxKKXMv/LAi3z/N3vL/uRps5rRDUaO98SmfrKo\nqO7+BG7n2GG+6SmG+SZSWeqrFu5SgJgZPredtctC6GqBaDxdsexTQS0yGEngc5pZ2SxB01QURaG9\nwU8sPnGz5PlMPo4tccd6YnzlJy9yqCsy8pjPbeVP37KOK9fWL8iMi9th4ZN/dBXf+tXukcGv9z92\ngIFYmj+74xKMZdS8uBw2ovE0fUNJqgOy3DMXYokMOoZR2aZyhvkWNQ2LASnyF0Api7y8KchQLM2J\n/gQms3nGRu8UNY1EMovJoLO6OTgrWa3FwmoxEa5amC0K5F95iRluMOnx+vnxo/v56ROHKGpnPoXd\nclkLf/SGtTjtC/tNx2hQ+OCtF1Hjd/L1h15G1+GxF7vI5ov85TsvG3cH1qv5PA5ODiawW00y42wO\ndPUn8LhGN618ooxhvslUlkbJNolXCXgd+Nx2+iMpeiIJzCYTToftnFpVqEWNZCqLAZ2mKhfeRTaL\nbbZU+51E4hlyeXVBBZ3z8kq/+/ArfKi6lWV1MgpjJqlqgU2bNtFxMkXd+tvpi54pzjNraf72T27i\novbKjhyYbbde3UbQY+MLP3wOtVhq4Pm57z7LJ957BVbz1Ol0v9fF4ZMxVrdIvcxsio6TbdI0nZ8+\nOfUwX10rSj8uMS6DQaEm6KLK7ySSyNA7lKJQ1LFYTNgs5kk7j+cLKtlcAbWgYrcYaap2yc/ZDGgN\n+3mlcwCL2bVgVjjm5TvBro4+/urLv+eSlTW87foVrG6p7NDFpaL71ACH0zUYl60YCZp0rUi8cxuu\nXCf1vtfP8RVWxpUXhPmkxcS939tOvlDkxYO93PvdZ/nrP7wSyxTBk8Gg4HE7ONQ1xJqWKmmkOEu6\n+uJ43KOXSHfsOzXSsX5NS3D8Yb7pLPVByTaJyRkMCkGvg6DXQb5QJJHOEU1kiaUyFNHRz9pUqyhg\nNhqwW03UBex4nbaylvtFecwmI821Hrr6k/g9C2Ms0rwMnIbtPNDLzgO9rGkJcsf1K1g/z7bBLxS6\nrvPkSyf4+kMvY/CvGHk8F+licO8vqfFZZ2Uq+Fxav6KGv/vA1fzTt58hk1PZdaiPf/n+dja994op\nCznNJiMWi4XOk0O0NQRn6YqXrvF20um6zv8+dmDk67dev2K8QykWVAJeWTYR5bOYjSNB1DBN01EU\n5P1mlvjddoZiGTK5writReabeRk4vfW65ew8aWYgmgFg79FB9h59hmV1Xu64fgVXXhCu+HiPxaJ3\nKMWWB3ex69CZfkZWs8Kpl35J8kRpjuHm/9ha8angUKoLyOdVCqqKpoOuaTC8y0UBdEBRMCgKitFQ\nag1gMc9YlmftshCfev9VfPab28jmi+w80Mu/3fccH3/P5VN+grTbLMQTGXoHkzIQuMJOjJNt2nWo\nj8PdUaDUhHV41+TZkukc1QGHvNmJ8yaZ5dnXXOfjlSP9WM2mef/9V2a7OdhkFEW5BHjhoUeeYvUF\n63jypS4eePzgmIGydUEnb71+Bddf3FRWke9SVCxq/HzbYX7wyD7yheLI4+vbAjz/y6/SfezMuIpK\nTQUvqEUy2TxqsYgRsFqMeBxW7DYTFpMRs6mUVRh+o9N1HU3TUYsaubxKKpsnnsqTLRRRDAYcNuuM\nFBDuOdLPP3zrmZHvy02XNnP3W9eX9YY7EEnQXu+VYvEKGYqnOTWYwfOqYttP/c8T7D06CMDH33M5\nV10QHntsJMEFrVWyjCLEAhVLZjnWE8fvm/0Pp7tfepFbb74WYIOu6zsne+68vcOYjAZuuKSZL/3F\nzXzivVfQFj4zIPHUYIqvPPAiH/7Xh/n50x1k8+ocXun803Eiwie+8hjf/tWekeAg6LVz921reP5n\nX6D7WAfhcJitW7cSDodHpoIPDAyc97nzBZVYIs1QNIGuFqgP2FnTHOSCtmqWNwapCbrwOG3YrGaM\nRsOoYEVRFIxGA1aLCY/LRl3Iw8rmEBe1VdNe78WsaAxFE8QTGTTt3AP+C1qr+OT7rhwZxfG754/x\nw9/tL+vYgNfF4e7YqGBUzAxd1znRl8D1qqB0b+fASNDUUOXmijX1Y47N5AoE3FJ7IsRC5nXZcNlN\nZHL5ub6USc3LpbqzGQwKV66t54o1dbx8uJ8HHjs4MpNsKJ7lGw/t5v5HD3Dr1W286apWXAt8G/35\niCayfP+3e/ndC8fOrIAp8KYr23jP61ZjNsKvKjAVvKhppNI5VFXF47DQUuPGabfM2JKJoig4bBaa\n6yw06TrRZJaT/UmKgNtpP6dZZOvaq/nonRv4wg+eA+BHv99P0GvjlsuWTXqcwaDgdtvp6Bpi9TLp\nLD6TBqNpjKaxafr7z6ptuuO1K8ZN46fTWVpkE4kQC15z7fCS3cyVacy0ebtUd+G69RM+72DXEA88\ndpAd+06NetxmMfH6K1p4yzXLCXiWzlJKQdV46JnD/Pj3+8nkzmTfmms93P3W9Sw/a/fRcB+nswvB\nBwYGzmkqeC6vkkpnsRgVaoJOfC77rP6gx1NZunoToBhwu2znFMT8/KkOvvHL3QAYFPjr913Jpavq\npjwulclhNyk01/mmfK6Ymqbp7Dnch883ekvyoRMRNn3lMQBq/A7+82O3jMkq5fIqiqbS1jB2l50Q\nYuGJJbMc603g987eLrvpLNXN+4zTeFY0Bvjr913JsZ44P33iIE++fAJN08nmVR58soOHth3hxg1N\n3H7dCmoDC2N747nQdZ3n9/fwzV/u5tRgauRxh9XEnTeu4tar28ZkY2ZiKng6kyebzeFxWljZQt2o\nSAAAIABJREFU6Mc2R7sgPE4ba1ttDEZLHYHtDit26/Qyjm+5tp2BeIafP9WBpsO/3vccm//k2nG3\nup/NabcSiaUYjKZlkOcM6I+kMFnMY4Lfs3fS3X7dinGX4pKpLKua/BW/RiHE7PC6bLhiaTK5/LTv\n6bNhQQZOw5prPfzFOy7lXTev5qdPHuL3LxyjoGqoRY3f7DjKI88d5dLVdbzpylYubKtaNMsquq6z\n58gAP/zdvpHaDygty918aQvvvmUNPpd1xs+bzubIZPJUeW201Ved0xJZJQR9DnxuG8d6okSiSbwe\n57QyX+9/wwVE4lmeevkE+UKRf/zWM/zzh64btyv12XweB8f7Ejjt5jkLHheDYlGjZyhFwD/6+32s\nJ86OvaWscsBj48YNTWOOVdUidotBvv9CLDLzecluQQdOw2oCTjbedjHvuHEVP3+6g4e3d5LJlba8\n79h7ih17T9FQ5eaNV7Xy2vWNC6JPxHgmCpig1BDwg2++iNb6mV86SmfzZDM5Ql4b7XXzc9eS0Wig\nNRwglszSeSo2rWnoBoPCR95+CdFklj1HBkik8/zTt5/lcx++ftKaOUVR8HmdHOwaYu2y+fl9WQhO\nDiSw28cG+g88fibbdNu1y8ftt5VIZWmt81T0+oQQs89oNNBS76WzJ0FgFpfsyrEga5ymkszk+fWz\nnfx6+xGG4tlRf2a3mrhuXSOvvaSJFY3+BZGF0nWd3Yf7+dHv948JmMIhF+++ZQ1XXTDzA3nzBZVE\nKkvAaaG+2jNvMkxTKahFOk4MgcGIy1F+rVsqW+BT//MEx3riAFy8vJq/ff/VU/YMy+YLKJpKuzTH\nnLZ8ocjeowMEX5VtOjWY5CNf+C2aDh6Hhf/+xOuxvSoQLmoaqWSGta1Vs3nJQohZ1HkyQkFXKr5k\nt+hrnKbislt4+w0ruf265ezYe4pfPnN4JODI5FQe3tHJwzs6CYdcXH9JE6+9uJHQPKxTyeVVnnip\ni4e2HeF4b3zUn4VDLu68cRXXXNQw481ANU0nlkxjNSqsbgosqOGLUOr0vao5xIneOJFYquwCQ6fN\nzCffdyWf+K/HiKfz7DrUx3d+vYcPvOnCSY+zWczEE6o0xzwHXb2l7OCrPfDYQYY7Trz5mvYxQRMM\nD/OV77cQi1lTjXfeLdktrHfEaTIZDVx9YZirLwzTeSrGr589whO7usid7sHTPZDk+7/Zy32/3cva\nZSEuX13Hpavr5rSgXNd1Dhwf4vcvHOPp3d2jdslBZQMmgEwuTyado7nWg28BT/xWFIXGWi+2SIru\ngQR+r6us/3TVficff88VfPrrT1HUdH72VAfNtR5uuKR50uM8bjunhkr1Ti7HzNeXLUbZXIFERiXo\nH50VPDWY5NEXjwPgsJl545VjW0Touo5WLMpUeiEWueElu6M9s7vLbjKLOnA627I6Lx9+63o+8KYL\neGbPSR7beZw9naWGj7oOe44MsOfIAF9/aDeN1W42rKrlslW1rGgMVLx2paBqHDw+xLN7T7L9lZMM\nxDJjnrOyKcCtV7dxVYXGzei6TjSWwu0w07aIui9X+Z1YLUYOd0fxeV1lLTeubQ3xJ29Zx5YHdwHw\n1Z/soj7kZmXT5Dvt/F4Xh7ujrFkWmnL+nYDOU9FxA58f/W7/SIPTP7i2Hec4dWaJVJZ6ye4JsSR4\nnDZctvmzy27JBE7D7FYzN25o5sYNzfRF0jz+4nEee/H4qO38XX0JuvoS/PSJQ9gsRlY0BljVHKQ1\n7KO1zkvQaz+veqJUtsCR7igd3RFe6Rxgb+cA2fzYTtQ2i4lrLgzz+iuW0d5Que3WubxKKpVZ8Fmm\niXicNlY3B9l/fAiv24GpjKDm9Vcs42hPjIe3d6IWNe793rN8/u4bCE4yQNZgUHA67XScGGJVszTH\nnEwkkaGoK2P+LU70xXnipS4A3A4Lb766bdzjiwVV2kAIsYTMp112Sy5wOlu138GdN67i7TespKsv\nwfP7e3h+/ykOHh8aqa/I5ou8fLiflw/3jxznsJmpCzqpCTgJee14nFa8TgtWswmz2YBBUSioGoWi\nRjpTIJrMEk3m6BlMcXIwyeA4GaVhJqPChW3VXHtRmKsuCI9b2zGTkuksaMVFnyWxWc2sbg5y4Ngg\nLrejrL/rXW++iO6+BHs6B4gmcnzuu8/yD//nOqzmiY+1WkwUVJUTvXEaa8+tE/tip2k6Xb1xvJ6x\nGaMf/G7/SNf7269bjsM2dgdsMp2jJuCUwFSIJcRoNNBc5531xpjjWdKB0zBFUWiq8dBU4+GO61cQ\nT+V44UAvuw72svfY4JhAJ50tcLg7OjKt/Xz53FbWtVVz8fJqLl1VO+7SRCVE4il8djONtQtjd+H5\nslpMrGoJsf/YAE6nHYt58h9/k9HA/33P5XziK4/RF0lzuDvK/zy4iz9/2yWTfr9cDhuRaBJXIoN/\nEWbwzlfPYAKzxTLmU2PnqRjbdncD4HVaeeOVreMen8vlCYUlKBViqZkvjTElcBqHx2nlhkuauOGS\nUsO9/miag8eH6DwVo/NUjJP9Cfqjac5lzqzLbqY+5KK13kd7g5/lDX4aqt2zGrhomk4kmqShykXI\nPz+K7WaLxVzacbf/2ACKa+rMk8dp5a//8Eo++d+PkysUeXTncZY3+HnDBG/qw3xeJ8d64jis5gW3\nK7GS8oUifdHMmPYDAD98ZN/I7+947Ypxs62ZbKkB62KpwRNCTM98WLKTO3oZqnwOqnwOrrmoYeSx\ngqrRH00TTWSJpXLEU3nyapGCqqFpOiaTAbPRgN1qwuey4nVZqfY7cM/xjquiphGJpmgLe/E4l848\nv7NZzEZWNgc5cHQQdxk1Ty11Xu6+Yz3//sPnAfj6Qy/TUudlVfPEfZsURcHrdnCwa5A1LYun2P58\nHTsVHbf9QMeJyMjsyYDHxusvH3/YciaTo7VO+jYJsVTNh8aYEjidI7PJQH3IRX1o4ezsUYsasXiK\nFY0+nON0al5KrGYTK1uC7Ds6iNfjnHK33WvWNdJxIsLPnz6MWtT5/Pd38Pk/u2HSYdImkxGrzcqR\nkxGWN0pzzGgiQ0bV8DvG3nZ+cFa26e2vXYllnDqyTK6A32VdMI1YhRCV4XHa8DqyZLJ57LbZX7KT\nO9ASoRY1YrEUq5uDSz5oGmY1m1jZGCAaS45sf5/MH73hAta2lAKgSCLLv963nYKqMTAwgKoWxj3G\nbrWQL8LJ/vi4f75UaJrO8d44XtfYnXD7jw2y82AvUMru3nRpy7ivkU5nqZ1ifqAQYmlorPaQzebK\nunfPNAmcloCiVso0rWpeeF3AK81uM9Me9jEUSzLV+CFdL5I48BB6odS6Yv+xIb5y/3buuusuNm3a\nNGHw5HHZ6YtmiSWz4/75UtDdF8dqG1sQDnDfb89km+68cSVm09jbUi6v4nGYx81ECSGWHqPRwLI6\nL9F4auonzzAJnBa54ULwlY0BmSA/AbfTxrJaD0PRyf8DRqMxOjv20/vCj9C1Ut+tx1/uIaoF6Og4\nTDQam/DYgM/JkZNRcnl1wucsVulsnsFkFodtbKZz16Fedh8ptfqoCzp57fqmcV8jlc4QrpJhvkKI\nM9xOG36nhXQ2N6vnlcBpEdN1naFYkuUNfuzj9MMRZ/jcduqDDqLx9ITPCYVCbNmyhSq3wtC+X488\nHlj7Jj712S8QCoUmPFZRFHweJweOD6IWtRm99vlM13UOd0fwuscWcWqazrd//crI1++8afW49Uv5\ngorLJrsThRBjNdR4yWXzFLXZu69K4LSIDcVStNR4ZHZamaoDLrx2E8nUxEtqtbW1bN68mVT3LpIn\nXgRAMZjY+nAHifTkn3pMJiN2u42OrsEplwUXi+7+OCazZdyA6PFdXRw9VcrStYV9XHvWrtWzJVNZ\nwlVS2ySEGMtgUGgL+4jEZm/JTgKnRSqWSFPrt+P3SAPG6Wis9WJSdDK5/Lh/3tPTwz333APA0L6H\nycVKDRv7Imm+8IPnKU5RqGizmtENRo73TLyst1hksgUG4tkxgbuqFjh5qpf7frt35LHbrmpE08Yu\nYxbUIk6rUZaZhRATctqtVHltJKf48DpTJHBahFKZHE6rkdqgfEqfLkVRaA37yefyFNTR8wMHBgbY\nuHEj3d3dhMNhtn7tfzD1PE0xlwTgpY6+UcHARFwOG/GMSu9gsiJ/h/lA03Q6uofGLNGpaoFNmzZx\n96f+Y2SY9doWH5//zF+NW2CfSEptkxBiauEqD0W1gKqOnfs60yRwWmTyBRWtoNJS55vrS1mwjEYD\nKxqDxOPpUevmPp+X9vY2wuEwW7ZsYd26dWz5r3+Hk08yPGDtgccP8sye7inP4fM4ODWUJpKYeG7h\nQnasJ4rVOrbnUjQao6OzGwIXAKAo8PIj36S7u3tMgb2qFrGbDVKfJ4SYkqIotIf9RBMT16nOFAmc\nFpGippFIZljeFFgSs+cqyWI20t7gI3rWTjuTycy9997L1q1bqa2tBUo1T1u//E+8/41rR5735ft3\n0tU7dd+mgK80liWVmd0dIZU2FEuTyKjjNqYLhUK85o6PYjCXGocmul6k+8grI8Ho2QX2iVSWxhqZ\nSSeEKI/NaqY+4CSerOwHUgmcFpFoLEV72Dfl/DVRHpfDSrjKRSQ+Onh69e65UCjEH7xmBa9ZVypu\nzuZV7v3edlLZ8fs6DVMUBZ/XyaETi6dNQa6gcrw3js8zttElwOHuCM/s6wNAK2SJdjwGwObNm0eC\nUSjVNtlMimSbhBDTUhN0YUCr6D1VAqdFIp7MUBtwyA66GVbld+K1m6csOlQUhQ+/dT3NtaV6nJMD\nSb784xem7GprNBjwuh0cODZIvlD5tflK0jSdQ11DeDzOcTOeuq6z9RcvD69qEjvyJFq+lFa/5557\n6OnpGXluIpmRbJMQ4py0hQMkk+mK7V6uWOCkKMrfKIrytKIoKUVRhip1HgHZXAGToksxeIU01XpB\nU6f8BGOzmNj03itwns6S7Nh3igcePzjl65tMRlxuBweODyzoHk+dJ4ewWCwTZjyfevkE+4+VbgWF\n1ACe4im2bt1KOBymu7ubjRs3MjAwQEEtYrcYJdskhDgnFrOR5jrvpH35zkclM05m4EfAVyt4jiWv\nqGlk0lnaGgJzfSmLlqIotDcESKcyUzZZqw26+Mt3XspwwuW+R/aOzGGbjNlkxG63s//YwgyeegYT\nZAtMOHAzlS3wjV/uHvnaMPAiW/77q6UC+y1bCIfDtLe34fN5S9mmatlJJ4Q4d363HZfNOGFrmfNR\nscBJ1/XP6Lr+JWD3lE8W5ywaS7Os3isT4yvMbDLS1uAjWkaTtUtW1vKum1YDpc12X/zhc/QMTX2c\n1WLCZrMtuOApmsjQM5TB4564Z9j3f7OXaKK03Hnpqhq2fmnz6AL7rVu599570XRFsk1CiBnRXOsj\nm8nN+P1U3m0XsGQ6S8hrxe20zfWlLAlOu5VwyFVW+vdtr13JZatLgUEyU+De7z5bVrHicPB04NjC\nGM2SSGXp7IkT8I0dqTLs0IkIv95+BACr2cif/sHF4xbYm0xmEqkMTTWSbRJCnD+j0UB7g5/YDHcV\nl8BpgVLVIppalOaAs6zK78RlM07ZQsBgUPjonZdSH3IBcKwnzld+8mJZxYpWiwmrzcq+o/3zumA8\nmyvQ0R0l4HVN2P6iqOls+emukYLwd968mirf+DvucnkVt80sXcKFEDPGYbNQF5zZFgXTCpwURfln\nRVG0SX4VFUVZMWNXJyYUS6Rpb/BLv6Y50Fzro1hQyRcmzyA5bWY2vfcKbKeH0z750gke2na4rHNY\nLSYcDjv7jw7My1YF2VyB/ceH8HldGAwT/wz+/OkOjpyMAtBc6+HNV7dN+NxkKkODZJuEEDOsJujC\npOgzdi+d7rjxfwW+McVzjpzjtYz4zN9twuMZvRX5tjvu5LY73nG+L70oxBMZ6kMumRY/RwwGheWN\nAfYeHcA/ReDQWOPhI2+/hM9/fwcA3/zVHpbV+VjbGprwmGEWswmD28G+owO0hn145smSbCZb4EDX\nEF6Pc9LauhN9iZERNIoCG2+7eMLnZ3IFfC4rVrP8TAshZl5bQ4BXjvRjMjn5xU/v58EHfjzqz+Px\n8ueHKpWe0q4oyvuBf9d1fcptX4qiXAK88NAjT3HhuvUVva6FKl9QUfN5VjZP/cYrKiuWytJ5Mk7Q\n75ryud/59R5+8sQhALxOK5//8xsIecsbwKxpOpFYkvqQi2r/xLVEsyGVyXGoK4LP58JomDhoKmo6\nf7vlcQ52RQB4yzVt/PGtF034/KFIgjXLQtK8VQhRMcl0jo7uKEH/2NY9u196kVtvvhZgg67rOyd7\nnUr2cWpUFGUd0AwYFUVZd/rX3N75FzBd10kkMyyr98/1pQjA67RR7bORSGWnfO57blnDRe1VAMRS\nOT7//e1l1y8ZDApBv5veSIbOk5Epm2pWymA0zcET0SmDJoBfPN0xEjTVBZ2855Y1Ez43nc0R8tok\naBJCVJTLYaU24DjveqdKFodvBnYCfw+4Tv9+J7Chgudc1GKJDI1VbixmeYOZL+qrPBjRyOYmH69i\nNBr42DsvGymMPtQV4cv3T91Z/Gw+j4OCprDnSB+ZKca5zCRd1+nqidE9mCJYRtB0rCfG989aovvz\nt10y4bKyrutks3lp3iqEmBW1QTdmRZ/ynj2ZSvZx+mNd143j/HqiUudczHJ5FatJITjBjiQxd9oa\nAmQy2SnbB3icVj7x3iuwng58n97dzfd+88q0zmW3WXC7nRzoGqK7L1bx7FMur7Kvc4BkrojfO/4o\nlVc//ws/eI6CWvpe3Hp1G6tbJl5WTqay1AVdGKUPmRBilrSWec+eiNytFgBd10km0yyr9831pYhx\nmIwGljcEiMan7hXSFvbxsXddxnA9+U+eOMTD2zunfb6g3008q7HnSB+xxMxPAtd1nZ7BBPuODWJ3\n2HCVWZj+jV/upqsvAUBLnZc/fN3aCZ+raTqqqk7YnkAIISph+J4di6fOaZ6dBE4LQPz0CAqpAZm/\n7DYzDWU2x7xsdR13vXndyNf//8928eye7mmf0+Ww4vW4ON6f4pUj/TMSQOm6zkAkxe7DfQwlCwT9\nbkxl/tw9s6eb3+w4CpQaXX7snZdNuqwcT5aaXUpLDSHEbLPbzDRWuc9pnp0ETvNcLq9iNiBLdAvA\ncHPMdHby5pgAb7yqldte0w6ApsMXfvg8L3f0TfucBoOCz+PA6bJzvD/Fyx29nOyPk5uix9Sr5Qoq\n3X0xdh/uozeWxed1Tasj/Ym+OF++/8xGlA+++SIaqieuW1LVIiaDjm+SMS1CCFFJQZ8Dn8NMMj31\nBp+zSdOUeay0iy7N2mVVc30pokzNtT72HR2gYDJNmSF83+svIJ7K8+jO46hFjc99dzuf+ZNrWd4w\n/V2TRoMBn8dR+pnJFug/NoRRAbvVhMdpxWY1YTQoGA2G0hKZppHLq8RSOTJZlaIONpsFv2/6Rdqp\nbIHPfXc72dPN5a69qIGbL22e9JhYMs2qRhlMLYSYW421Xg4cG6Cglj+lQQKneSyRzNIgu+gWFINB\nob3Rz76jg1M2xzQYFO5+63qSmTzP7eshm1fZ/PWn+fRd19IWPrd6NkVRcNgtOOwWoJTZ6Y/n0bRM\naeyJroOiYFAUFIMBu9WM13vujTU1TedLP3qekwNJoNQd/O471k+6/JbJFfA5rTJaRQgx5xRFob0x\nyEsvTb1SMEyW6uapfEHFZNCpmuOGh2L6rGYTrfVeImUMljQaDXzsXZezdllp51kqW+AzX39qZEzJ\n+TKZjLidVrxuBz6PA5/Xic/jwOO243Zay65fmsi3f72H5/f3AOCym9n0h1eOjJiZSDqdpaFaRqsI\nIeYHk9FAU4136ieeJoHTPJVIZmiRXXQLlsdpo8ZvK6vRmtVs5G/+6CpWNwcBSGYKfHrrUxzunpng\nqVIefPIQP3uqAwCDAn/17supDUwe6CdSWeqDk49qEUKI2TadEWZy95qH4skM9UGnzO1a4OpCHixG\nyOTyUz7XbjXxqQ9cxcqmUt1PMlPgnq89yZ4j/ZW+zHPy+IvH+dav9ox8vfH2i1nXXj3pMWpRQy8W\nJYsqhFjQJHCaZwpqEQMa1YGp55+J+a+13k8umyur8NBuNfN3H7h6JPOUyal89pvb2L73ZKUvc1q2\n7e7mP//3zA66d928mlsuWzblcbFEitZ6n7QfEEIsaBI4zTPxRJpWmUW3aBiNBlY0BYnHU2V1+XbY\nzNzzx1ezYWUNAAVV4/Pf287Pnjp0To3aZtrjLx7nCz/YQfH03+UNVyzjzhtWTnlcJpsn4LRit0lB\nuBBiYZPAaR5JJDPUBhzTWmsV85/VbKI17GMolizv+RYTm/7wSq6/uBEo9Xn65i/38J//u3NaW2Zn\n2sPbO/mP+19gOP676dJm7nrLuikzSJqmk83kCEtBuBBiEZDAaZ5Q1SK6plEjS3SLksdpoz7gJJYo\nr0utyWjgI2/fMCqb8+jO43zqf56kZ2jq3XozqVjU2PqLl9ny4C6Gk15vuGIZH759PcZJ2i0Mi8VT\nNNd5ZR6dEGJRkDvZPBFLpGkLS/3HYlYTdOG0GkllyusXYjAovPuWNaXRJabSf9VDJyL81Zd/z+Mv\nHq/kpY5IpHN89lvbeGjb4ZHHbntNO3/6B+sm7VE1LJsv4LKb8LrOvVeUEELMJxI4zQOJVJZqn10a\nAi4BLXU+9KJKLl/+SJRr1zXwjxuvG9nqn8mpfOnHL/C57zxLX2T6c5bKtWPfKf7iS7/j5Y7Szj6T\nUeHDb72Y97/xwrICfE3TSaeyNNdKWw0hxOIhgdMcU9UiWlGlLjT9URdi4VEUheWNQdLpDOo06pXa\nwn7+7SM38Nr1jSOP7dh3io9+8RHuf/QAmdyZQExVCwwMDIw6fmBgAFUtlHWuoXiGL/34eT73nWeJ\nJkrZMY/Twqc/eG1Zu+eGxeIpWmSJTgixyMgdbY7FEmna6v2yRLeEmIwGVjYFiSXSZe20G2a3mvno\nnZfysXddhs9lBSBfKPL93+7lQ59/mB//fj/RRIpNmzZx11130dNT6ujd09PDXXfdxaZNmyYNnmLJ\nHN/85W7u/tff8PiLXSOPX7Kyhn/78xtZc7q7eTnS2Rxep0WW6IQQi45s35pDyXSWKq9NtmgvQVaL\nifawj0PdUYI+17QC52svamD9ihp+8Nt9/OrZw2g6JNJ57ntkHz9+dD+FbCNDhV42fvjP2fzpv+Oe\ne+6hu7sbgGg0Rih0JgDK5FRePtzHYzuP88KBHtTimUDOYTPzwVsv5IZLmqZ1fapaJJ/Ls3zZ5A0x\nhRBiIVLmQ2+YYYqiXAK88NAjT3HhuvVzfTkVpapF0uksa5aFJNu0hEXiGY71Jgj6z203ZVdvnP99\n/CBPvdTFq5NXuq5TSPVTiPdgtyjccdutuN1uUpkCsWSOwyejHO+JjTnOYjLwxqtauf01K/CezmxN\nx2AkwaqmgNTsCSEWjJ07d7JhwwaADbqu75zsuZJxmiOxRJpVTQEJmpY4v8dOoajRM5TC753+KJLG\nGg//3zsu5V03reKhbUd46uUTxFKluiRFUbC4qrG4Spmfnz49+U48n9vKay9u4s3XtBHw2Kf/lwGi\n8TT1IZcETUKIRUsCpzmQlF104izVfieqWmQokcbrdpzTa9QGXdz1lov4wJsu4PHnD/If3/gJqsmL\n2VWDYhi/lNGgQFOtl+UNfq5YW8+6tqrzKuROZXK4bEaqZRadEGIRk8BplhXUInqxSF1IxqqIM+qr\nPBSLMeLJDB7XuWV7ACKRIb70T5vo7u4mHA7zt/d8ms/e+0UGIglCtQ3c/eEPU18TwmU3Uxt0YbfO\nzC0gX1DRCirN4fILyIUQYiGSXXWzLBZP09Ygu+jEWI21XtxWI/Fk5pxfw+fz0t7eRjgcZsuWLVy6\nYT1b/uNeQo4i7dUmbrq8nQvbqlhW75uxoKmoaSSSGZY3BcpqiimEEAuZZJxmUTyRIVzlkll0YkJN\ndT6OnYqSSGVxO6e/ld9kMnPvvfeO2j1XW1vL1q1b8fm8mEwzuzysaTrRWIqVjQHMJuOMvrYQQsxH\nknGaJbm8ismgS/2HmFJznQ+XxVD2XLtXM5nMo1oOAIRCoRkPmnRdJxJL0lrvlZYaQoglQwKnWaDr\nOqlUhtaw1DWJ8jTV+fA7zURiszvQdzqGYimaqt14ziEzJoQQC5UETrMgGkvRXOuRpQwxLeFqL7V+\nO4ORJPOp3xrAUDRJOOgk4D23XYBCCLFQSeBUYcOjJ3zuc98pJZau6oCLllo3g5EkalGb68tB13UG\nIgkaQi6qZNlZCLEESeBUQcOjJxprvHN9KWIB87ntrG4OkEikyOXVqQ+okKKmMRRJ0lLjIeiTTJMQ\nYmmSwKlCdF0nlkizvEG2aIvzZ7OaWbOsimIhf17tCs5VvqASjaVob/DhP8eu4kIIsRhI4FQhsUSG\nBhk9IWaQyWhgZXOIkNvKYCSBqhZn5bzJdI5sJsealhAux/Rn1wkhxGIiDYUqIJPN47AYCEkNiKiA\nmqALr8vKke4oumLA7bJVpKFqUdOIxlME3TbCDSHJnAohBBI4zThVLZLL5Vi7rHquL0UsYjarmTWt\nVQxEUpwYSGK1WmYsG6TrOolUFq2osqLBj8NmmZHXFUKIxUACpxk0XNe0ujkon87FrAj5nfg9dvqj\nKXojCUwmEy6H7Zx+/oqaRiqdo6iqhENuAl67jAYSQohXkcBpBkXiaVpqPTJSRcwqo9FAbdBNtd/F\nUCxNfzRNrqhjMZuwWc2T9g9T1SLZfIF8voDFaKA24CTgkYBJCCEmIu/wMySRzBB0S78mMXcMBoWQ\n30nI7yRfKJJI54incsTjGVRdBw1QFNB1UMCoKNhtJqo8VjxOLxazNGgVQoipSOA0AzK5PGYDhKs8\nc30pQgBgMRsJeh0EX9XZu1jUMBplM60QQpwruYOep8LpJpetYb8sb4h5T4ImIYQ4P3IVJQjgAAAL\neklEQVQXPQ9FTSORSLOiMShvSEIIIcQSIO/250jXdSLRUidlqQ0RQgghlgYJnM7RYDRJa70Xp106\nKQshhBBLhQRO5yASTdIQcuF12eb6UoQQQggxiyRwmqZIPEWVz0aVjFMRQgghlhwJnKYhlkgTdFmo\nC0nbASGEEGIpksCpTLFEGq/dTLjaO9eXIoQQQog5Ig0wyxCNp/G7zDRI0CSEEEIsaRI4TSESS1Hl\ntcrynBBCCCEqs1SnKEqzoihfUxTliKIoaUVRDimK8mlFUcyVOF8l6LrOYCRJrd8uQZMQQgghgMpl\nnFYBCvCnwGHgAuBrgAP4RIXOOWOKmkY0lqKl1iNDe4UQQggxoiKBk67rDwMPn/XQUUVR/hX4EPM8\ncMoXVBLJDCsa/Thslrm+HCGEEELMI7NZ4+QDhmbxfNOWTOegqLKmJSRjVIQQQggxxqwEToqitAN/\nDnxsNs43XbquE42l8DktNDaGUBRlri9JCCGEEPPQtIrDFUX5Z0VRtEl+FRVFWfGqY8LAr4Af6rr+\n9Zm8+JmQy6sMRZI01bhpqvNJ0CSEEEKICSm6rpf/ZEUJAsEpnnZE13X19PPrgUeBbbqu/3EZr38J\n8MLlV12DxzO6Z9Jtd9zJbXe8o+xrnYqm6cSTGWwmhWVhPyaj9AIVQgghFrv77ruP++67b9RjsViM\nJ554AmCDrus7Jzt+WoHTdJzONP0eeA54n17GiYYDp4ceeYoL162vyHUBpDI58rk8TTWya04IIYRY\n6nbu3MmGDRugjMCpIjVOpzNNjwGdlHbRVQ8vgem63luJc5YjkyuQSWcJeW3UhasxGGRZTgghhBDl\nq1Rx+C1A6+lfXacfUwAdmPXtaulMnkw2R8BlZdmyEGaT7JgTQgghxPRVqo/Tt4BvVeK1y1XUNFLp\nHKqqEvLYaK2TgEkIIYQQ52dRzapTixqZbJ58voDNYqQu4MDnssuSnBBCCCFmxIIOnApqkVy+QKFQ\nRNc1HFYTNT47XpdPdskJIYQQYsbNy8AplcmTSGUp7cPT0XUdXQdd19A0HYMBTAYDdquJKo8Vl92C\n1WKSHkxCCCGEqKh5GThVe63UB0ptAgyKgmJQMBoUTEYjJqNBlt6EEEIIMSfmZeDkdtqkv5IQQggh\n5h0pBBJCCCGEKJMETkIIIYQQZZLASQghhBCiTBI4CSGEEEKUSQInIYQQQogySeAkhBBCCFEmCZyE\nEEIIIcokgZMQQgghRJkkcBJCCCGEKJMETkIIIYQQZZLASQghhBCiTBI4CSGEEEKUSQInIYQQQogy\nSeAkhBBCCFEmCZyEEEIIIcokgZMQQgghRJkkcBJCCCGEKJMETkIIIYQQZZLASQghhBCiTBI4CSGE\nEEKUSQInIYQQQogySeAkhBBCCFEmCZyEEEIIIcokgZMQQgghRJkkcBJCCCGEKJMETkIIIYQQZZLA\nSQghhBCiTBI4CSGEEEKUSQInIYQQQogySeAkhBBCCFEmCZyEEEIIIcokgZMQQgghRJkkcBJCCCGE\nKJMETkIIIYQQZZLASQghhBCiTBI4CSGEEEKUSQInIYQQQogySeAkhBBCCFEmCZyEEEIIIcokgZMQ\nQgghRJkkcBJCCCGEKNP/a+9eQ+WqzjCO/59YbfAKVUlsjcZbrVaaaFWUYFHUaKxtrZiqSWm0pSBR\nalFahSKxovWuqAheP0SpafRDqa1gxNZCTTReQgpCTEAbUau2SWus16jn9cPaB8bTM3P2rDl7Zs/M\n84MhzM7ae7/zsufsd9asWauywknSHyS9IukDSf+UdJ+kPao63zBYtmxZr0Poa85f55zDzjh/nXMO\nO+P8da7KHqe/APOBrwKnA/sBD1V4voHnC74zzl/nnMPOOH+dcw474/x17gtVHTgibml4+qqka4Df\nS9omIj6t6rxmZmZmVenKGCdJXwIWAitdNJmZmVm/qrRwknSNpHeBTcAM4LQqz2dmZmZWpba+qpN0\nNXBJiyYBHBQRG4rn1wH3AHsDS4D7gVNb7D8VYN26de2ENTS2bNnCmjVreh1G33L+Ouccdsb565xz\n2Bnnb3wNdcfUidoqIkofWNKuwK4TNHs5Ij4ZZ9+vAK8CR0fE6ibHXwD8tnRAZmZmZpNnYUQ80KpB\nWz1OEbEZ2JwZzDbFv19s0WYFaSzURuDDzPOYmZmZtWMqMJNUh7TUVo9TWZKOBI4AngT+C+wPXAHs\nDhwSER9P+knNzMzMKlbV4PD3SXM3PQ68CNwNrAWOddFkZmZm/aqSHiczMzOzQeS16szMzMxKcuFk\nZmZmVpILp5qQdL6kfxSLIj8t6YgJ2h8r6XlJH0raIGlRt2Ktq3ZyKOn7kh6T9C9JWyStkjS3m/HW\nUbvXYcN+cyR9LGmoJ4jJeB9vJ+kqSRuL9/LLks7pUri1lJHDhZLWSnqvWFD+3mK1iqEj6RhJD0t6\nXdKIpO+W2Mf3kja5cKoBSWcCN5ImCT0U+DuwQtJuTdrPBP4E/BmYBdwC3CPpxG7EW0ft5hD4FvAY\nMA84DHgC+KOkWV0It5Yycji63y7AUtKPQYZWZv4eAo4DziUtiH42sL7iUGsr42/hHNK1dzdwMHAG\ncCRwV1cCrp8dSD/EWkyakLol30vyeHB4DUh6GlgdERcWz0WaLPTWiLhunPbXAvMi4hsN25YBu0TE\nKV0Ku1bazWGTY7wA/C4irqwu0vrKzWFx7W0ARoDvRcRh3Yi3bjLexycDDwD7RsTbXQ22pjJyeDFw\nXkQc0LDtAuCXEbFXl8KuJUkjwGkR8XCLNr6XZHCPU49J2hb4JqniByBSNfs4cHST3Y7i/z/dr2jR\nfqBl5nDsMQTsBPynihjrLjeHks4F9gF+XXWMdZaZv+8AzwGXSHpN0npJ10uacMmHQZSZw6eAGZLm\nFceYBswHHqk22oHhe0kGF069txtpVvW3xmx/C5jeZJ/pTdrvLKnVzOyDKieHY/2C1M394CTG1U/a\nzqGkA4DfkJYoGKk2vNrLuQb3BY4Bvk5aAP1C0ldNt1cUY921ncOIWAX8EFguaSvwBmnS5QsqjHOQ\n+F6SwYWTDb1ijcTLgPkRsanX8fQDSVNI60ouiYiXRjf3MKR+NIX09eaCiHguIh4FLgIW+aZVjqSD\nSeNyLieNVTyJ1AN6Zw/DsgHX1lp1VolNwKfAtDHbpwFvNtnnzSbt34mIjyY3vL6Qk0MAJJ1FGkh6\nRkQ8UU14faHdHO4EHA7MljTaQzKF9K3nVmBuRPy1oljrKOcafAN4PSLebdi2jlSA7gm8NO5egysn\nh5cCKyPipuL5C5IWA3+T9KuIGNubYp/ne0kG9zj1WLEEzfPA8aPbivE2xwOrmuz2VGP7wtxi+9DJ\nzCGSzgbuBc4qPu0PrYwcvgMcAswm/RpnFnAHaYmlWcDqikOulcxrcCXwZUnbN2w7kNQL9VpFodZW\nZg63Bz4Zs22E9Isy94BOzPeSHBHhR48fwA9I6/v9CPgaqZt5M7B78f9XA0sb2s8E/gdcS/pDuxjY\nCpzQ69fSRzlcUOTsPNInrNHHzr1+Lf2Sw3H2XwKs6fXr6Jf8kcbUvQIsBw4iTZGxHrij16+lj3K4\nCPioeB/vA8wBngFW9fq19Ch/O5A+uMwmFZA/L57PaJI/30syHv6qrgYi4sFinpIrSDfvtcBJEfHv\nosl0YEZD+42Svg3cDPyM9On0JxExtPPotJtD4Kekgai38/nBuEuBH1cfcf1k5NAaZLyP3yvmy7kN\neJZUICwnjbcbShk5XCppR+B84AbgbdKv8i7tauD1cThpTrooHjcW20f/rvleMgk8j5OZmZlZSR7j\nZGZmZlaSCyczMzOzklw4mZmZmZXkwsnMzMysJBdOZmZmZiW5cDIzMzMryYWTmZmZWUkunMzMzMxK\ncuFkZmZmVpILJzMzM7OSXDiZmZmZleTCyczMzKykzwCifwduERx+7wAAAABJRU5ErkJggg==\n", "text/plain": [ "" ] }, "metadata": {}, "output_type": "display_data" } ], "source": [ "m.optimize_restarts(10) # try 10 different starting locations \n", "m.plot()" ] }, { "cell_type": "markdown", "metadata": {}, "source": [ "The fit chosen is the one with the highest maximum likelihood. This is not always what we want, and so we may choose to use methods such as looking at prediction error on a held out dataset, or cross-validation to choose the hyper parameters. " ] }, { "cell_type": "markdown", "metadata": {}, "source": [ "We can also do things like fix certain parameters, constrain them to lie in a given range, and put prior distributions over them. Notice how this information is reported when you type `print(m)`." ] }, { "cell_type": "code", "execution_count": 28, "metadata": { "collapsed": false }, "outputs": [ { "name": "stdout", "output_type": "stream", "text": [ "Warning: changing parameters to satisfy constraints\n", "WARNING: reconstraining parameters GP_regression.rbf.lengthscale\n", "WARNING: reconstraining parameters GP_regression.rbf.variance\n", "\n", "Name : GP regression\n", "Objective : 12.012149258332974\n", "Number of Parameters : 3\n", "Number of Optimization Parameters : 2\n", "Updates : True\n", "Parameters:\n", " \u001b[1mGP_regression. \u001b[0;0m | value | constraints | priors \n", " \u001b[1mrbf.variance \u001b[0;0m | 1.42852320245 | +ve | Ga(3.2, 2.1)\n", " \u001b[1mrbf.lengthscale \u001b[0;0m | 0.200000007687 | 0.2,1.0 | \n", " \u001b[1mGaussian_noise.variance\u001b[0;0m | 0.1 | +ve fixed | \n" ] }, { "name": "stderr", "output_type": "stream", "text": [ " /Users/pmzrdw/GPy/GPy/core/parameterization/priors.py:301: RuntimeWarning:divide by zero encountered in log\n" ] }, { "data": { "image/png": "iVBORw0KGgoAAAANSUhEUgAAAgUAAAFkCAYAAACw3EhvAAAABHNCSVQICAgIfAhkiAAAAAlwSFlz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\nzAxwUmBmZmbZv//KqMul33eQAAAAAElFTkSuQmCC\n", "text/plain": [ "" ] }, "metadata": {}, "output_type": "display_data" } ], "source": [ "m.Gaussian_noise.variance.fix(0.1)\n", "m.rbf.lengthscale.constrain_bounded(lower=0.2, upper=1)\n", "\n", "gamma_prior = GPy.priors.Gamma.from_EV(1.5, 0.7)\n", "gamma_prior.plot()\n", "m.rbf.variance.set_prior(gamma_prior)\n", "m.optimize()\n", "print(m)" ] }, { "cell_type": "markdown", "metadata": {}, "source": [ "To remoe these constraints and priors, we can do something like the following:" ] }, { "cell_type": "code", "execution_count": 29, "metadata": { "collapsed": false }, "outputs": [ { "name": "stdout", "output_type": "stream", "text": [ "\n", "Name : GP regression\n", "Objective : 11.016765214370487\n", "Number of Parameters : 3\n", "Number of Optimization Parameters : 3\n", "Updates : True\n", "Parameters:\n", " \u001b[1mGP_regression. \u001b[0;0m | value | constraints | priors\n", " \u001b[1mrbf.variance \u001b[0;0m | 1.42852320245 | | \n", " \u001b[1mrbf.lengthscale \u001b[0;0m | 0.200000007687 | | \n", " \u001b[1mGaussian_noise.variance\u001b[0;0m | 0.1 | | \n" ] } ], "source": [ "m.unconstrain()\n", "m.rbf.variance.unset_priors()\n", "print(m)\n", "m.constrain_positive() # we've removed the postive constraint above, so we should reset this constraint.\n" ] }, { "cell_type": "markdown", "metadata": {}, "source": [ "### Exercise 6:\n", "Generate some data from the model $$y = x*sin(x)+e \\qquad e \\sim N(0,0.1^2)$$ for x in the range $[0,5]$. Fit a GP regression model. Do your hyperparameter estimates look reasonable? Try constraining the noise variance to be $0.1^2$. Predict the function in the range $[-5,10]$. Do your predictions look reasonable? What could you do to fix this?" ] }, { "cell_type": "code", "execution_count": null, "metadata": { "collapsed": true }, "outputs": [], "source": [] } ], "metadata": { "anaconda-cloud": {}, "kernelspec": { "display_name": "Python [conda root]", "language": "python", "name": "conda-root-py" }, "language_info": { "codemirror_mode": { "name": "ipython", "version": 3 }, "file_extension": ".py", "mimetype": "text/x-python", "name": "python", "nbconvert_exporter": "python", "pygments_lexer": "ipython3", "version": "3.5.2" }, "widgets": { "state": { "18254e5c823e4e98b16a1fa1ba2f0601": { "views": [ { "cell_index": 74 } ] }, "1f7845e1830243379232e8c4e97eaa41": { "views": [ { "cell_index": 67 } ] } }, "version": "1.2.0" } }, "nbformat": 4, "nbformat_minor": 0 }
indexrbf.lengthscaleconstraintspriors
[0] 0.20000000 +ve